You are on page 1of 81

§Ò C¦¥NG ¤N THI TèT NGHIÖP M«n to¸n

N¨m häc 2009 - 2010

CẤU TRÚC ĐỀ THI NGHIỆP THPT NĂM 2010


I. PHẦN CHUNG CHO TẤT CẢ THÍ SINH (7,0 điểm)
Câu Nội dung kiến thức Điểm

• Khảo sát, vẽ đồ thị của hàm số.


• Các bài toán liên quan đến ứng dụng của đạo hàm và đồ thị của hàm số: Chiều
I
biến thiên của hàm số. Cực trị. Tiếp tuyến, tiệm cận (đứng và ngang) của đồ 3,0
thị của hàm số. Tìm trên đồ thị những điểm có tính chất cho trước; tương
giao giữa hai đồ thị (một trong hai đồ thị là đường thẳng);...

• Hàm số, phương trình, bất phương trình mũ và lôgarit.


II • Giá trị lớn nhất và nhỏ nhất của hàm số.
3,0
Tìm nguyên hàm, tính tích phân.
• Bài toán tổng hợp.

Hình học không gian (tổng hợp): Tính diện tích xung quanh của hình nón tròn
III xoay, hình trụ tròn xoay; tính thể tích khối lăng trụ, khối chóp, khối nón tròn 1,0
xoay, khối trụ tròn xoay; tính diện tích mặt cầu và thể tích khối cầu.

II. PHẦN RIÊNG (3,0 điểm)


Thí sinh học chương trình nào thì chỉ được làm phần dành riêng cho chương trình đó
(phần 1 hoặc phần 2).

1. Theo chương trình Chuẩn:


Câu Nội dung kiến thức Điểm
Phương pháp toạ độ trong trong không gian:
− Xác định toạ độ của điểm, vectơ.

− Mặt cầu.
IV.a 2,0
− Viết phương trình mặt phẳng, đường thẳng.

− Tính góc; tính khoảng cách từ điểm đến mặt phẳng. Vị trí tương đối của
đường thẳng, mặt phẳng và mặt cầu.

1
• Số phức: Môđun của số phức, các phép toán trên số phức. Căn bậc hai của

V.a số thực âm. Phương trình bậc hai hệ số thực có biệt thức ∆ âm. 1,0
• Ứng dụng của tích phân: Tính diện tích hình phẳng, thể tích khối tròn xoay.

2. Theo chương trình Naââng cao:


Câu Nội dung kiến thức Điểm
Phương pháp toạ độ trong trong không gian:
− Xác định toạ độ của điểm, vectơ.

− Mặt cầu.
IV.b − Viết phương trình mặt phẳng, đường thẳng. 2,0

− Tính góc; tính khoảng cách từ điểm đến đường thẳng, mặt phẳng; khoảng
cách giữa hai đường thẳng. Vị trí tương đối của đường thẳng, mặt phẳng và
mặt cầu.
• Số phức: Môđun của số phức, các phép toán trên số phức. Căn bậc hai của
số phức. Phương trình bậc hai với hệ số phức. Dạng lượng giác của số phức.

ax 2 + bx + c
• Đồ thị hàm phân thức hữu tỉ dạng y = và một số yếu tố liên
px + q
V.b 1,0
quan.
• Sự tiếp xúc của hai đường cong.

• Hệ phương trình mũ và lôgarit.


• Ứng dụng của tích phân: Tính diện tích hình phẳng, thể tích khối tròn xoay.

2
phÇn chung cho tÊt c¶ c¸c thÝ sinh
CHñ §Ò kh¶o s¸t, vÏ ®å thÞ hµm sè
vµ c¸c bµi to¸n liªn quan

i. kh¶o s¸t vµ vÏ ®å thÞ hµm sè


1. D¹ng 1: Hµm bËc ba y = ax3 + bx2 + cx + d ( a ≠ 0 )
1.1. C¸c b−íc kh¶o s¸t vµ vÏ ®å thÞ.

1. TËp x¸c ®Þnh: D = R


2. Sù biÕn thiªn
* Ta cã y’ = 3ax2 + 2bx + c
- XÐt dÊu y’ tõ ®ã suy ra sù ®ång biÕn, nghÞch biÕn cña hµm sè
* T×m cùc trÞ.
- T×m cùc trÞ tøc lµ t×m c¸c ®iÓm cùc ®¹i vµ cùc tiÓu cña hµm sè (nÕu cã)
- C¸ch t×m:
+ NÕu t¹i x = x0 mµ y’ ®æi dÊu tõ (+) sang (-) th× hµm sè ®¹t cùc ®¹i t¹i x = x0 vµ gi¸ trÞ
cùc ®¹i lµ yC§ = y(x0)
+ NÕu t¹i i x = x0 mµ y’ ®æi dÊu tõ (-) sang (+) th× hµm sè ®¹t cùc tiÕu t¹i x = x0 vµ gi¸ trÞ
cùc tiÓu lµ yCT = y(x0)
L−u ý: NÕu qua x0 mµ y’ ®æi dÊu th× hµm sè ®¹t cùc trÞ t¹i x0, ng−îc l¹i x0 kh«ng lµ cùc
trÞ cña hµm sè.
* T×m c¸c giíi h¹n:
b c d
+ lim ( ax 3 + bx 2 + cx + d ) = lim {ax 3 (1 + + 2 + 3 )}
x →+∞ x →+∞ ax ax ax
+∞, nÕu a > 0
=
−∞, nÕu a < 0
b c d
+ lim ( ax 3 + bx 2 + cx + d ) = lim {ax 3 (1 + + 2 + 3 )}
x →−∞ x →−∞ ax ax ax
+∞, nÕu a < 0
=
−∞, nÕu a > 0

* LËp b¶ng biÕn thiªn.

3
3. VÏ ®å thÞ:
Khi vÏ ®å thÞ hµm sè ngoµi c¸c chó ý ®· tr×nh bµy trong SGK häc sinh cÇn l−u ý thªm
mét sè ®iÓm sau c¸c b−íc sau:
- BiÓu diÔn c¸c ®iÓm cùc trÞ (nÕu cã) lªn hÖ trôc to¹ ®é.
- T×m giao ®iÓm cña ®å thÞ víi c¸c trôc to¹ ®é, c¸c ®iÓm ®Æc biÖt vµ biÓu diÔn chóng
lªn hÖ trôc to¹ ®é.

1.2. VÝ dô: Kh¶o s¸t sù biÕn thiªn vµ vÏ ®å thÞ hµm sè y = -x3 + 3x2 – 4
1.3. H−íng dÉn

1. TËp x¸c ®Þnh: D = R


2. Sù biÕn thiªn
* Ta cã y’ = -3x2 + 6x
y’ = 0 ⇔ x = 0, x = 2
XÐt dÊu y’ (b¶ng xÐt dÊu nµy häc sinh cã thÓ lµm ngoµi giÊy nh¸p)
x -∞ 0 2 +∞
y - 0 + 0 -

Tõ b¶ng xÐt dÊu y ta cã
Hµm sè nghÞch biÕn trªn c¸c kho¶ng (- ∞ ; 0) vµ (2; + ∞ )
Hµm sè ®ång biÕn trªn kho¶ng (0; 2)
* Cùc trÞ:
Hµm sè ®¹t cùc tiÓu t¹i x = 0, yCT = y(0) = -4
Hµm sè ®¹t cùc ®¹i t¹i x = 2, yC§ = y(2) = 0
* C¸c giíi h¹n:
3 4
lim (-x 3 + 3x 2 - 4) = lim {-x 3 (1 - + )} = −∞
x →+∞ x →+∞ x x3
3 4
lim (-x 3 + 3x 2 - 4) = lim {-x 3 (1 - + 3 )} = +∞
x →−∞ x →−∞ x x
* B¶ng biÕn thiªn.

x -∞ 0 2 +∞
y - 0 + 0 -
+∞
0
y

-4
-∞

4
3. VÏ ®å thÞ:
- Giao ®iÓm cña ®å thÞ víi c¸c trôc to¹ ®é 4

Giao víi Ox (-1; 0), (2; 0)


2

Giao víi trôc Oy (0; -4)


2
-1 3

Chän x = -2, y = 16 -5 O 5

X = 3, y = -4 -2

-4

-6

1.4. Bµi tËp tù gi¶i:


Kh¶o s¸t vµ vÏ ®å thÞ c¸c hµm sè sau:

1. y = x3 + 3x2 - 4 4. y = -2x3 + 5 7. y = x3 – 3x2


2. y = -x3 +3x – 2 5. y = x3 + 4x2 + 4x 8. y = –x3 + 3x2 – 2
3. y = x3 + x2 + 9x 6. y = x3 – 3x + 5 9. y = x3 – 6x2 + 9

2. D¹ng 2: Hµm trïng ph−¬ng y = ax4 + bx2 + c ( a ≠ 0 )


2.1. C¸c b−íc kh¶o s¸t vµ vÏ ®å thÞ.

1. TËp x¸c ®Þnh: D = R


2. Sù biÕn thiªn
* Ta cã y’ = 4ax3 + 2bx = 2x(2ax2 + b)
- XÐt dÊu y’ tõ ®ã suy ra sù ®ång biÕn, nghÞch biÕn cña hµm sè
* T×m cùc trÞ.
C¸ch t×m cùc trÞ hµm bËc bèn ®−îc lµm t−¬ng tù nh− hµm bËc ba
* T×m c¸c giíi h¹n:
b c +∞, nÕu a>0
+ lim ( ax 4 + bx 2 + c) = lim {ax 4 (1 + + )} = 
x →±∞ x →±∞ ax 2 ax 4 −∞, nÕu a<0

* LËp b¶ng biÕn thiªn.


3. VÏ ®å thÞ: Khi vÏ ®å thÞ hµm sè bËc bèn häc sinh cñng cÇn l−¬u ý mét sè ®iÓm nh− vÏ ®å thÞ
hµm bËc ba.

5
2.2. VÝ dô: Kh¶o s¸t sù biÕn thiªn vµ vÏ ®å thÞ hµm sè y = x4 - 2x2 + 2
2.3. H−íng dÉn

1. TËp x¸c ®Þnh: D = R


2. Sù biÕn thiªn
* Ta cã y ‘ = 4x3 - 4x = 4x(x2 - 1)
y’ = 0 ⇔ x = 0, x = 1, x = -1
B¶ng xÐt dÊu y’
x -∞ -1 0 1 +∞
4x - - 0 + +
2
x -1 + 0 - - 0 +
y’ - 0 + 0 - 0 +
Tõ b¶ng xÐt dÊu y’ ta cã
Hµm sè ®ång biÕn trªn c¸c kho¶ng (-1; 0) vµ (1; + ∞ )
Hµm sè nghÞch biÕn trªn c¸c kho¶ng (- ∞ ; -1) vµ (0; 1)
* Cùc trÞ:
Hµm sè ®¹t cùc ®¹i t¹i x = 0, yC§ = y(0) = 2
Hµm ®¹t cùc tiÕu t¹i x = ± 1, yCT = y( ± 1) = 1
* Giíi h¹n:
2 2
x →±∞
{
+ lim ( x 4 − 2 x 2 + 2) = lim x 4 (1 − 2 + 4 )} = + ∞
x →±∞ x x

* B¶ng biÕn thiªn

x -∞ -1 0 1 +∞
y’ - 0 + 0 - 0 +
+∞ +∞
2
y
1 1
3. §å thÞ f( x) = ( x4-2⋅x2)+2 6

Giao ®iÓm cña ®å thÞ víi trôc Oy: (0; 2)


4

-5 -1 1 5

-2

-4

6
2.4. Bµi tËp tù gi¶i:
Kh¶o s¸t vµ vÏ ®å thÞ c¸c hµm sè sau:

1. y = -x4 + 8x2 - 1 4. y = − x 4 + 2 x 2 + 3 7. y = x4 – 2x2


4 2
2. y = -x4 – 2x2 + 3 5. y = −
x4
− x2 +
3 8. y = x + x + 1
1 4 2 3 2 2 1 4 1 2
3. y = x +x − 9. y = x + x +1
2 2 1 4 3 4 2
6. y = x − 3x 2 +
2 2
ax + b
3. D¹ng 3: Hµm ph©n thøc h÷u tû B1/B1 y = (ac ≠ 0)
cx + d
3.1. C¸c b−íc kh¶o s¸t vµ vÏ ®å thÞ.

 d
1. TËp x¸c ®Þnh: D = R \ − 
 c

2. Sù biÕn thiªn
ad − cb
* Ta cã y′ =
(cx + d )2
d d
- NÕu ad – cb > 0 th× hµm sè ®ång biÕn trªn c¸c kho¶ng ( −∞; − ) vµ ( − ; + ∞ )
c c
d d
- NÕu ad – cb < 0 th× hµm sè nghÞch biÕn trªn c¸c kho¶ng ( −∞; − ) vµ ( − ; + ∞ )
c c
* Hµm sè kh«ng cã cùc trÞ
L−u ý: Lo¹i hµm sè nµy kh«ng cã cùc trÞ
* T×m c¸c giíi h¹n:
ax + b a ax + b d
lim = , limd = ∞ , do ®ã ®å thÞ hµm sè nhËn c¸c ®−êng th¼ng x = −
x →±∞ cx + d c x →− cx + d c
c
a
lµm tiÖm cËn ®øng vµ y = lµm tiÖm cËn ngang.
c
ax + b −∞, nÕu ad – cb < 0
lim − =
x → −  cx + d
 d +∞, nÕu ad – cb > 0
 c

ax + b −∞, nÕu ad – cb > 0


lim =
 d
x → − 
+
cx + d +∞, nÕu ad – cb < 0
 c

* LËp b¶ng biÕn thiªn.


3. VÏ ®å thÞ:
Khi vÏ ®å thÞ hµm sè b1/b1, ngoµi c¸c l−u ý trong SGK häc sinh cÇn l−u thªm mét sè ®iÓm sau:
- VÏ c¸c ®−êng tiÖm cËn lªn hÖ trôc to¹ ®é
- T×m giao ®iÓm cña ®å thÞ víi c¸c trôc to¹ ®é, c¸c ®iÓm ®Æc biÖt vµ biÓu diÔn chóng lªn
hÖ trôc t¹o ®é.
7
3.2. VÝ dô
−x + 2
Kh¶o s¸t sù biÕn thiªn vµ vÏ ®å thÞ hµm sè y =
2x +1
3.3. H−íng dÉn

 1
1. TËp x¸c ®Þnh D = R \ − 
 2

2. Sù biÕn thiªn
5
* Ta cã y′ = − 2
< 0, ∀x ∈ D
( 2 x + 1)
1 1
Do ®ã hµm sè lu«n nghÞch biÕn trªn c¸c kho¶ng (−∞; − ) vµ ( − ; + ∞ )
2 2
* Hµm sè kh«ng cã cùc trÞ
* Giíi h¹n
−x + 2 1 −x + 2 −x + 2
lim = − ; lim − = − ∞; lim + =+∞
x →±∞ 2x +1 2 x → − 1  2 x + 1 x → −  2 x + 1
 1
 2  2

1
Do ®ã ®ß thÞ hµm sè nhËn c¸c ®−êng th¼ng x = − lµm tiÖm cËn ®øng vµ ®−êng th¼ng y
2
1
=− lµm tiÖm cËn ngang.
2
* B¶ng biÕn thiªn

x 1
-∞ - +∞
2

y′ - -

1 +∞
y -
2 1
-∞ -
2
3. §å thÞ -x+2 6
f( x) =
2⋅x+1

1
- O
2
-5 1 5
-
2

-2

Giao ®iÓm cña ®å thÞ víi trôc Ox: (2; 0)


Giao ®iÓm cña ®å thÞ víi trôc Oy: (0; 2) -4

8
3.4. Bµi tËp tù gi¶i
Kh¶o s¸t vµ vÏ ®å thÞ c¸c hµm sè sau:

x+2 x −1 1− 2x 2x + 3 1− x
1. y = − 3. y = 5. y = 7. y = 9. y =
x +1 x +1 2x − 4 2− x 1+ x
x−2 x+3 x −5 x+3
2. y = 4. y = 6. y = 8. y =
2x +1 x −1 1− x x +1

Ii. Mét sè d¹ng to¸n liªn quan ®Õn bµi to¸n kh¶o s¸t hµm sè
4. D¹ng 4: Dùa vµo ®å thÞ biÖn luËn theo tham sè m sè nghiÖm cña ph−¬ng tr×nh F(x;m) =0 (1).
4.1. C¸ch giải:

Bµi to¸n nµy th−êng ®i kÌm theo sau bµi to¸n kh¶o s¸t vµ vÏ ®å thÞ hµm sè y = f(x) v× thÕ
®Ó sö dông ®−îc ®å thÞ hµm sè võa vÏ tr−íc hÕt ta biÕn ®æi ph−¬ng tr×nh (1) t−¬ng ®−¬ng: f(x) =
g(m).
Khi ®ã sè nghiÖm cña ph−¬ng tr×nh (1) lµ sè giao ®iÓm cña ®å thÞ hµm sè y = f(x) vµ
®−êng th¼ng y = g(m).
Dùa vµ ®å thÞ, ta suy ra kÕt qu¶ biÖn luËn vÒ sè nghiÖm cña ph−¬ng tr×nh (1).

4.2. VÝ dô: Cho hµm sè y = -x3 + 3x2 – 4


a/ Kh¶o s¸t vµ vÏ ®å thÞ hµm sè
b/ Dùa vµ ®å thÞ biÖn luËn theo tham sè m sè nghiÖm cña ph−¬ng tr×nh: -x3 + 3x2 - 4 - m = 0 (1)
4.3. H−íng dÉn:

a/ ViÖc kh¶o s¸t vµ vÏ ®å thÞ hµm sè ®· ®−îc tr×nh bµy (xem bµi 1.2).
b/ Ph−¬ng tr×nh (1) t−¬ng ®−¬ng: -x3 + 3x2 - 4 = m(2).
Sè nghiÖm cña ph−¬ng tr×nh (1) lµ sè giao ®iÓm cña ®å thÞ hµm sè y = -x3 + 3x2 - 4 vµ
®−êng th¼ng y = m (lu«n song song hoÆc trïng víi trôc Ox).
Dùa vµo ®å thÞ (h×nh 4.3) ta cã: 4 y=m

* Khi m<-4 hoÆc m>0: Ph−¬ng tr×nh (1) v« nghiÖm


2

* Khi m = 0 hoÆc m = -4: Ph−¬ng tr×nh (1) cã hai nghiÖm -5 5

y=m

* Khi -4<m<0: Ph−¬ng tr×nh (1) cã ba nghiÖm ph©n biÖt.


-2

-4

y= m
-6

H×nh 4.3

9
4.4. Bµi tËp tù gi¶i:
1. Cho hµm sè y = x3 + 4x2 + 4x
a/ Kh¶o s¸t sù biÕn thiªn vµ vÏ ®å thÞ hµm sè
b/ Dùa vµo ®å thÞ biÖn luËn theo tham sè m sè nghiÖm cña ph−¬ng tr×nh x3 + 4x2 + 4x + 2 – m =
0(1)
2. Cho hµm sè y = y = x3 – 3x + 5
a/ Kh¶o s¸t sù biÕn thiªn vµ vÏ ®å thÞ hµm sè
m
b/ Dùa vµo ®å thÞ biÖn luËn theo tham sè m sè nghiÖm cña ph−¬ng tr×nh x3 – 3x + 5 + = 0(1)
3
x4 3
3. Cho hµm sè y = − − x2 +
2 2
a/ Kh¶o s¸t sù biÕn thiªn vµ vÏ ®å thÞ hµm sè
x4
b/ Dùa vµo ®å thÞ biÖn luËn theo tham sè m sè nghiÖm cña ph−¬ng tr×nh − − x 2 + 1 + m = 0(1)
2
1 4 3
4. Cho hµm sè y = x − 3x 2 +
2 2
a/ Kh¶o s¸t sù biÕn thiªn vµ vÏ ®å thÞ hµm sè
1 4 3
b/ Dùa vµo ®å thÞ biÖn luËn theo tham sè m sè nghiÖm cña ph−¬ng tr×nh x − 3x 2 + + m =
2 2
0(1)
5. Cho hµm sè y = x3 – 3x2
a/ Kh¶o s¸t sù biÕn thiªn vµ vÏ ®å thÞ hµm sè
b/ T×m c¸c gi¸ trÞ cña m ®Ó ph−¬ng tr×nh sau cã 3 nghiÖm ph©n biÖt: x3 – 3x2 – 3 + m = 0(1)
5. D¹ng 5: Bµi t−¬ng giao gi÷a ®−êng th¼ng y = px + q vµ ®å thÞ hµm sè y = f(x).
5.1. C¸ch gi¶i:

Sè giao ®iÓm cña ®−êng th¼ng y = px + q víi ®å thÞ hµm sè y = f(x) lµ sè nghiÖm cña
ph−¬ng tr×nh hoµnh ®é giao ®iÓm: f(x) = px + q(1)
Nh− vËy ®Ó xÐt sù t−¬ng giao cña ®−êng th¼ng vµ ®å thÞ hµm sè ta gi¶I vµ biÖn luËn
ph−¬ng tr×nh (1).
Dùa vµ sè nghiÖm cña ph−¬ng tr×nh (1) ta kÕt luËn vÒ sù t−¬ng giao cña ®−êng th¼ng y =
px + q víi ®å thÞ hµm sè y = f(x).

10
x+3
5.2. VÝ dô Cho hµm sè y = (C). Chøng minh r»ng víi mäi gi¸ trÞ cña m, ®−êng th¼ng (d): y
x +1
= 2x+m lu«n c¾t (C) t¹i hai ®iÓm ph©n biÖt.
5.3. H−íng dÉn

x+3
Ta cã ph−¬ng tr×nh hoµnh ®é giao ®iÓm: = 2x+m (1).
x +1
§−êng th¼ng (d) lu«n c¾t (C) t¹i hai ®iÓm ph©n biÖt víi mäi m khi vµ chØ khi ph−¬ng tr×nh (1) lu«n
cã hai nghiÖm ph©m biÖt víi mäi m.
ThËt vËy
x+3  x + 3 = (2 x + m)( x + 1)
= 2x+m ⇔ 
x +1  x ≠ −1
 g ( x) = 2 x 2 + (m + 1) x + m − 3 = 0 (2)
⇔
 x ≠ −1
XÐt ph−¬ng tr×nh (2), ta cã:
∆ = m 2 − 6m + 25 > 0
 ∀m . VËy ph−¬ng tr×nh (1) lu«n cã hai nghiÖm kh¸c -1. Do ®ã
 g (−1) = −2 ≠ 0
®−êng th¼ng (d) lu«n c¾t (C) t¹i hai ®iÓm ph©n biÖt víi mäi m.

5.4. Bµi tËp tù gi¶i.


x +1
1. Cho hµm sè y = (C). CMR ®−êng th¼ng 2x-y+m=0 lu«n c¾t ®å thÞ (C) t¹i hai ®iÓm ph©n
x −1
biÖt thuéc 2 nh¸nh cña (C.)
x+3
2. T×m m ®Ó ®−êng th¼ng y = x +m c¾t ®å thÞ (C): y = t¹i hai diÓm ph©n biÖt.
x −1
3 − 2x
3. Cho hµm sè y = .T×m tÊt c¶ c¸c gi¸ trÞ cña tham sè m ®Ó ®−êng th¼ng y = mx+2 c¾t ®å
x −1
thÞ hµm sè ®· cho t¹i hai ®iÓm ph©n biÖt.
6. D¹ng 6: ViÕt ph−¬ng tr×nh tiÕp tuyÕn t¹i mét ®iÓm M(x0, y0) thuéc ®å thÞ y = f(x).
6.1. C¸ch gi¶i

* Ph−¬ng tr×nh tiÕp tuyÕn cña ®å thÞ t¹i ®iÓm M(x0, y0) thuéc ®å thÞ cã d¹ng:
y-y0 = f’(x0)(x-x0) (1)
* T×m f’(x0) thay vµo (1) ta ®−îc tiÕp tuyÕn cÇn t×m.

6.2. VÝ dô Cho hµm sè y = x3 – 3x + 5. ViÕt PTTT cña ®å thÞ t¹i ®iÓm M(1; 3).

11
6.3. H−íng dÉn:

* Ph−¬ng tr×nh tiÕp tuyÕn cña ®å thÞ t¹i ®iÓm M(1, 3) thuéc ®å thÞ cã d¹ng:
y-y0 = f’(x0)(x-x0) (1)
* Ta cã y’ = f’(x) = 3x-3
⇒ f’(1) = 0 thay vµo (1) ta ®−îc PTTT cÇn t×m lµ: y = 3

6.4. Bµi tËp tù gi¶i:


1. Cho hµm sè y = − x 4 + 2 x 2 + 3 . ViÕt PTTT cña ®å thÞ t¹i ®iÓm M(2, 3)
x4 3
2. Cho hµm sè y = − − x 2 + ViÕt PTTT cña ®å thÞ t¹i ®iÓm M(1, 0)
2 2
1 4 3
3. Cho hµm sè y = x − 3 x 2 + . ViÕt PTTT cña ®å thÞ t¹i ®iÓm M(1, -2)
2 2

4. Cho hµm sè y = x3 – 3x2 ViÕt PTTT cña ®å thÞ t¹i c¸c giao ®iÓm cña nã víi trôc Ox.
5. Cho hµm sè y = –x3 + 3x2 – 2. ViÕt PTTT cña ®å thÞ t¹i ®iÓm M(2, 2)
1− 2x
6. Cho hµm sè y = ViÕt PTTT cña ®å thÞ t¹i giao ®iÓm cña nã víi trôc Ox.
2x − 4
x −5
7. Cho hµm sè y = ViÕt PTTT cña ®å thÞ t¹i giao ®iÓm cña nã víi trôc Ox
1− x
7. D¹ng 7: TÝnh diÖn tÝch h×nh ph¼ng ®−îc giíi h¹n bëi ®å thÞ hµm sè y = f(x) vµ c¸c ®−êng th¼ng
x = a, x = b, trôc Ox.
7.1. C¸ch gi¶i:

b
* Ta cã diÖn tÝch S = ∫ f ( x) dx .
a

§Ó tÝnh S ta ph¶I ph¸ dÊu trÞ tuyÖt ®èi cña biÓu thøc d−íi dÊu tÝch ph©n, muèn vËy ta lµm nh−
sau:
C¸ch 1: LËp b¶ng xÐt dÊu f(x), tõ ®ã ta cã thÓ ph¸ dÊu trÞ tuyÖt ®èi.
C¸ch 2: NÕu trªn kho¶ng (a; b) ®å thÞ hµm sè y = f(x) n»m phÝa trªn trôc hoµnh th× f ( x) = f ( x )
Ng−îc l¹i, nÕu ®å thÞ n¨mg phÝa d−íi trôc hoµnh th× f ( x) = − f ( x ) .
Sau khi ph¸ dÊu trÞ tuyÖt ®èi ta tÝnh tÝch ph©n b×nh th−êng, kÕt qu¶ ®ã chÝnh lµ diÖn tÝch cÇn t×m.

f(x) = x3 -4⋅x 6

7.2 VÝ dô: Cho hµm sè y = x3 - 4x.


4

a/ Kh¶o s¸t vµ vÏ ®å thÞ hµm sè.


b/ TÝnh diÖn tÝch h×nh ph¼ng giíi h¹n bëi
2

O 1
®å thÞ hµm sè víi c¸c ®−êng x = -1, x = 2 -5
-1 5

7.3 H−íng dÉn. -2

12 -4

H×nh 7.3
a/ B¹n ®äc tù gi¶i, ®å thÞ (h×nh 7.3)

b. C¸ch 1
2
* Ta cã diÖn tÝch cÇn t×m S = ∫ x 3 − 4 x dx .
−1

* Ph¸ dÊu trÞ tuyÖt ®èi: §Æt f(x) = x3 - 4x = x(x2 - 4)


Trªn kho¶ng (-1; 2), ta cã x3 - 4x = 0 ⇔ x = 0, x = 2.
* LËp b¶ng xÐt dÊu f(x).
x -1 0 2
x - 0 +
x2 -4 - -4 -
f(x) + 0 -
Tõ b¶ng xÐt dÊu, ta cã
2 0 2 0 2
S = ∫ x 3 − 4 x dx = ∫ x3 − 4 x dx + ∫ x 3 − 4 x dx = ∫ ( x 3 − 4 x)dx − ∫ ( x 3 − 4 x)dx
−1 −1 0 −1 0
0 2
= ∫ ( x 3 − 4 x)dx − ∫ ( x3 − 4 x)dx
−1 0

TÝnh kÕt qu¶ trªn ta suy ra diÖn tÝch cÇn t×m.


C¸ch 2: Tõ ®å thÞ cña hµm sè (h×nh 7.3), ta cã:
Trªn kho¶ng (-1; 0) ®å thÞ n»m phÝa trªn trôc hoµnh vµ trªn kho¶ng (0; 2) ®å thÞ n»m phÝa d−íi
trôc hoµnh, nªn ta cã:
2 0 2 0 2
S = ∫ x 3 − 4 x dx = ∫ x3 − 4 x dx + ∫ x 3 − 4 x dx = ∫ ( x 3 − 4 x)dx − ∫ ( x 3 − 4 x)dx
−1 −1 0 −1 0
0 2
= ∫ ( x 3 − 4 x)dx − ∫ ( x3 − 4 x)dx
−1 0

TÝnh kÕt qu¶ trªn ta suy ra diÖn tÝch cÇn t×m.

7.4. Bµi tËp tù gi¶i


TÝnh diÖn tÝch h×nh ph¼ng giíi h¹n bëi:
1. y = x3 – 3x2 vµ c¸c ®−êng th¼ng x = -1, x = 2, trôc Ox.
2. y = –x3 + 3x2 – 2 vµ c¸c ®−êng th¼ng x = -1, x = 2, trôc Ox
3. y = x3 – 6x2 + 9 vµ c¸c ®−êng th¼ng x = -2, x = 1, trôc Ox
4. y = − x 4 + 2 x 2 + 3 vµ c¸c ®−êng th¼ng x = 0, x = 1, trôc Ox
x4 3
5. y = − − x 2 + vµ c¸c ®−êng th¼ng x = -1, x = 1, trôc Ox
2 2
1 4 3
6. y = x − 3 x 2 + vµ c¸c ®−êng th¼ng x = -2, x = 1, trôc Ox
2 2
13
8. D¹ng 8: T×m §iÒu kiÖn cña tham sè m ®Ó ®å thÞ hµm bËc ba y = ax3 + bx2 + cx + d
a/ Cã cùc trÞ.
b/ Lu«n ®ång biÕn hoÆc nghÞc biÕn trªn R.
8.1. C¸ch gi¶i:

a/ * T×m tËp x¸c ®Þnh D = R


* TÝnh y’ = 3ax2 + 2bx + c
Hµm sè cã cùc trÞ (cùc ®¹i vµ cùc tiÓu) khi vµ chØ khi ph−¬ng tr×nh y’ = 0 cã hai nghiÖm
ph©n biÖt.
⇔ ∆ y′ > 0 ⇒ m cÇn t×m
b/ * T×m tËp x¸c ®Þnh D = R
* TÝnh y’ = 3ax2 + 2bx + c
Hµm sè lu«n ®ång biÕn trªn R khi vµ chØ khi
∆ y ′ ≤ 0
y′ ≥ 0, ∀x ∈ R ⇔  ⇒ m cÇn t×m
a y′ > 0
Hµm sè lu«n nghÞch biÕn trªn R khi vµ chØ khi
∆ y′ ≤ 0
y′ ≥ 0, ∀x ∈ R ⇔  ⇒ m cÇn t×m
a y′ < 0

14
CHỦ ĐỀ PHƯƠNG TRÌNH
BẤT PHƯƠNG TRÌNH MŨ VÀ LÔGARIT
A. PHÖÔNG TRÌNH MUÕ
Kieán thöùc cô baûn
1 – Caùc tính chaát cuûa luyõ thöøa.

1
1.1 a0 = 1, a1 = a , a− n = (a ≠ 0)
an
am
1.2 am .an = am + n , n
= am − n
a
n m n
1.3 (a ) = ( am ) = am.n
n
n an  a 
1.4 an b n = ( a.b ) , = 
bn  b 
m
1.5 a n = n am

2– Caùc tính chaát cuûa haøm soá muõ.

Cho haøm soá y = ax ( 0 < a ≠ 1)


2.1 Taäp xaùc ñònh D = R.
2.2 Taäp giaù trò : T = (0; +∞).
2.3 Haøm soá y = ax ñoàng bieán khi a > 1 vaø nghòch bieán khi 0 < a < 1.
2.4 ax = a t ⇔ x = t
a > 1 0 < a < 1
2.5  x t
⇒x>t ;  x t
⇒x<t
a > a a > a

3 – Phöông phaùp giaûi phöông trình muõ.


3.1- Phöông trình muõ ñôn giaûn nhaát.
* ax = a b ⇔ x = b ( 0 < a ≠ 1)
* ax = b ⇔ x = loga b ( 0 < a ≠ 1, b > 0 )
x
Ví dụ 3 = 5 ⇔ x = log35
3.2 Phöông trình muõ thöôøng gaëp
a. Phöông phaùp ñöa veà cuøng moät cô soá.
a f ( x ) = a g( x ) ⇔ f ( x ) = g ( x ) ( 0 < a ≠ 1)
Ví duï: 2 = 8 ⇔ 2 = 2 ⇔ x = 3
x x 3

Baøi taäp: Giaûi caùc phöông trình sau


x x −1
1
1)   = 5 2) 5.5x – 5x = 2x+1 + 2x+3 3) 5 .8 x x
= 500
5
15
4) 16-x = 82(1-x) 5) 52x = 625
b. Phöông phaùp ñaët aån soá phuï
Ñaët t = ax (t > 0) {choïn cô soá a thích hôïp}
Ví dụ: Giải pt : 4 x − 3.2 x + 2 = 0
Giải .
Biến đổi pt 4 x − 3.2 x + 2 = 0 ⇔ (2 2 ) x − 3.2 x + 2 = 0 ⇔ (2 x ) 2 − 3.2 x + 2 = 0 (1) .
• Đặt t=2x , đk t>0 .
t = 1
• Pt (1) ⇔ t 2 − 3t + 2 = 0 ⇔  .
t = 2
• Với t=1 ⇒ 2 x = 1 ⇔ 2 x = 20 ⇔ x = 0 .
• Với t=2 ⇒ 2 x = 2 ⇔ 2 x = 21 ⇔ x = 1
Đáp số : Nghiệm của phương trình là x=0 , x=1 .
Baøi taäp: Giaûi caùc phöông trình
1) 8.3x + 3.2 x = 24 + 6 x 2) 4 x +1 + 2 x + 4 = 2 x + 2 + 16
2 2
3) 34x + 8 − 4.32x + 5 + 27 = 0 4) 2 x −x
− 22+ x −x = 3
2 2
5) 101+ x − 101− x = 99
c) Phöông phaùp laáy loâragit (cô soá thích hôïp) hai veá.
af ( x ) = b g ( x ) ( 0 < a ≠ 1, 0 < b ≠ 1)
Laáy loâgarit cô soá a ta ñöôïc:
f ( x ) = g ( x ) log a b
2
Ví dụ: Giải pt 3x.2 x = 1 .
Giải
Lấy Lôgarit cơ số 3 hai vế , ta được :
2 2 2
PT 3x.2 x = 1 ⇔ log 3 (3x.2 x ) = log 3 1 ⇔ log 3 (3x.2 x ) = 0
2
⇔ log 3 3x + log 3 2 x = 0 ⇔ x + x 2 log 3 2 = 0 ⇔ x (1 + x log 3 2) = 0
x = 0
x = 0 x = 0 
⇔ ⇔ ⇔
1 + x log 3 2 = 0  x log 3 2 = −1  x = −1 = − log 2 3 = log 2 1
 log 3 2 3

Baøi taäp: Giaûi caùc phöông trình


3( 2 x −1)
2
1) 3 .2
x x +1
= 72 2) 2 x = 3 x −1 ,
x −1 1
4)logx+1(x2 + 3x - 1) = 1
4
3) 5 x8 x
= 500 5) x ln x = 10 x

16
B. PHÖÔNG TRÌNH LOÂGARIT
Kieán thöùc cô baûn
Cho a > 0, a ≠ 1 ; x > 0, x1 > 0, x2 > 0 .

1) Ñònh nghóa
log a x = b ⇔ x = a b

Chuù yù:
(1) x = aloga x ( ∀x > 0 )
(2) x = log a ax ( ∀x ∈ R )
2) Tính chaát
1) log a a = 1, log a 1 = 0
2) log a ( x1.x2 ) = log a x1 + log a x2
x1
3) log a = log a x1 − log a x2
x2
4) log a xα = α log a x , ∀α ∈ R
log b x
5) log a x = ( 0 < b ≠ 1)
log b a
1 1
Chuù yù: log a b = ; log aα x = log a x , α ≠ 0
log b a α

3) Phöông phaùp giaûi


a) Ñöa veà cuøng moät cô soá
+ log a f ( x ) = b ⇔ f ( x ) = ab
f ( x ) > 0 ( hoaëc g ( x ) > 0 )
+ log a f ( x ) = log a g ( x ) ⇔ 
f ( x ) = g ( x )

AÙp duïng:
Giải pt : log 2 ( x + 3) + log 1 (2 x + 1) = 0
2
Giải
1
ĐK: x > −
2
log 2 ( x + 3) + log 1 (2 x + 1) = 0 ⇔ log 2 ( x + 3) − log 2 (2 x + 1) = 0
2

⇔ log 2 ( x + 3) = log 2 (2 x + 1)

⇔ x + 3 = 2x + 1

⇔ x=2

17
Chuù yù: Khi khoâng söû duïng coâng thöùc töông ñöông nhôù ñaët ñieàu kieän ñeå haøm soá
loâgarit coù nghóa (cô soá phaûi lôùn hôn 0 vaø khaùc 1, bieåu thöùc laáy loâgarit phaûi döông).
Baøi taäp Giaûi caùc phöông trình
1) log 3 ( x 2 + 2x ) = 1 2) log 3 x + log3 ( x + 2 ) = 1

1 2
3) lg ( x 2 + 2x − 3 ) = lg ( x − 3 ) 4) log 2 ( x − 1) + log 1 ( x + 4 ) = 0
2 2

1 1 8
5) log 2
( x + 3) + log 4 ( x − 1) = log 2 ( 4x ) 6) log 2 2 + log 2 4x = 3
2 4 x

b) Ñaët aån soá phuï


Choïn aån soá phuï thích hôïp, bieán ñoåi phöông trình ñaõ cho thaønh moät phöông
trình ñaïi soá.
AÙp duïng:
Giải pt : log 3 (2 x + 1) = 2 log 2 x +1 3 + 1
Giải
1
ĐK: − < x ≠ 1
2
2
log 3 ( 2 x + 1) = 2 log 2 x +1 3 + 1 ⇔ log 3 ( 2 x + 1) − −1 = 0
log 3 ( 2 x + 1)

Đặt t = log3(2x - 1)
2
Ta được t − − 1 = 0 ⇔ t2 - t - 2 = 0 ⇒ t = -1, t = 2
t
1 1
Với t = -1 ⇒ log3(2x - 1) = -1 ⇔ 2x + 1 = 3-1 = ⇔x= −
3 3

t = 2 ⇒ log3(2x - 1) = 2 ⇔ 2x + 1 32 = 9 ⇔ x = 4

Baøi taäp: Giaûi caùc phöông trình sau


1). log 3 x + log x 9 = 3 . 2). log 22 x − 3.log 2 x + 2 = 0 .

3). Lg4(x - 1)2 + lg2(x - 1)3 = 25 4).log3(2x + 1) = 2log2x+1 + 1


5) 2log4(3x - 2) + 2log3x-2 = 5

18
C. BÊt ph−¬ng tr×nh mò
1. sö dông tÝnh ®¬n ®iÖu
1) 2x < 3x/2 + 1 x<2(chia cho2x)
2) 2.2x + 3.3x > 6x -1 . x < 2 (chuyÓn 1 sang tr¸i vµ chia hai vÕ cho 6x)
3) 8x + 18x ≤ 2.27X →x≥0
2. §−a vÒ cïng c¬ sè
1) 2.14x + 3.49x – 4x ≥ 0 → x ≥ log2/73 (chia hai vÕ cho 49x vµ ®Æt t = (2/7)x)
2) 2x + 2x + 1 ≤ 3x + 3x – 1 →x≥2
2 1
+1
1x 1x
3) 96]   + 3  > 12 → -1<x < 0
3 3
x − x −1
x2 −2 x 1
4) 3 ≥  →x≥2
3
5) 3x + 1 – 22x + 1 - 12x/2 < 0 → x > 0(chia cho 3x vµ ®Æt Èn phô t =
( 4 / 3 )x)
2 .3 x − 2 x + 2
6) ≤ 1 →0<x≤ log3/23 (chia c¶ tö vµ mÉu cho 2x)
3x − 2 x
x −1
7) ( 5+2 )
x −1
≥ ( 5−2 ) x +1
→x ≥1,-2≤ x<-1

19
Chñ ®Ò Nguyªn hµm - tÝch ph©n

A. nguyªn hµm
I. KiÕn thøc c¬ b¶n

1, §Þnh nghÜa nguyªn hµm:


F(x) ®−îc gäi lµ nguyªn hµm cña hµm sè f(x) trªn kho¶ng (a;b) nÕu:
F'(x)=f(x), ∀x ∈ ( a; b )

2, TÝnh chÊt cña nguyªn hµm:


'
1. ( ∫ f ( x)dx ) = f ( x)
2. ∫ af ( x)dx = a ∫ f ( x)dx (a ≠ 0)

3. ∫ [ f ( x) ± g ( x)] dx = ∫ f ( x)dx ± ∫ g ( x)dx


4. ∫ f (t )dt = F (t ) + C ⇒ ∫ f (u ( x))u '( x)dx = F (u( x)) + C .
3, B¶ng c¸c nguyªn hµm:

Nguyªn hµm cña c¸c hµm sè s¬ cÊp th−êng gÆp Nguyªn hµm cña c¸c hµm sè hîp (u=u(x))

∫ dx = x + C ∫ du = u + C
xα +1 uα +1
∫ x dx = ∫ u dx =
α α
+ C (α ≠ −1) + C (α ≠ −1)
α +1 α +1
dx du
∫ x
= ln x + C ( x ≠ 0) ∫ u
= ln u + C ( x ≠ 0)

∫ e dx = e ∫ e du = e
x x u u
+C +C

ax au
∫ a dx = ∫ a du =
x u
+ C (0 < a ≠ 1) + C (0 < a ≠ 1)
ln a ln a

∫ cos xdx = sin x + C ∫ cos udu = sin u + C


∫ sin xdx = − cos x + C ∫ sin udu = − cos u + C
dx du
∫ cos 2
x
= tgx + C ∫ cos 2
u
= tgu + C

dx du
∫ sin 2
x
= − cot gx + C ∫ sin 2
u
= − cot gu + C

20
II. C¸c d¹ng to¸n c¬ b¶n
1. D¹ng 1.. ¸p
¸ dông c«ng thøc biÕn ®æi

1. ∫ af ( x)dx = a ∫ f ( x)dx ( a ≠ 0)

2. ∫ [ f ( x) ± g ( x)] dx = ∫ f ( x)dx ± ∫ g ( x)dx

1.1 VÝ dô
a) ∫ (2x 2 − 3x + 5)dx = ∫ 2x 2dx - ∫ 3xdx + ∫ 5dx

2x 3 3x 2
= 2∫ x 2dx - 3∫ xdx + 5∫ dx = − + 5x + C
3 2

b) ∫  3 sin x − 2 dx = 3∫ sin xdx − 2∫ 2 dx = –3cosx – 2tgx + C


2 1
 cos x  cos x

3 1 1
1 2 1
x 4 + 2 x 3 + 3x 2 − − −
c) ∫
x
dx = ∫x 4
dx + 2∫ x 3
dx + 3∫ x 2
dx

1 2 1
− +1 − +1 − +1 3 1 1
3
x 4
x x 2
4 4
= +2 +3 +C = x + 6x 3 + 6x 2 + C
1 1 1 3
− +1 − +1 − +1
4 3 2

1.2 Bµi tËp tù gi¶i


T×m nguyªn hµm c¸c hµm sè sau:
1 1
a, f(x)= x3 + cos x − b, f(x)= 5 x 4 − 2 x +
x cos 2 x
3 2 3x 4 − 2 x3 + 5
c, f(x)= − + + 5sin x d, f(x)= (x ≠ 0),
x4 x x2
2. D¹ng 2. ¸p dông nguyªn hµm ®æi biÕn sè d¹ng 1
2.1. D¹ng : I = ∫ (ax + b) n dx

a) C¸ch gi¶I tæng qu¸t


§Æt u = ax + b
b) VÝ dô : T×m I = ∫ (5 x + 3)5 dx
1 1
Ta cã (5 x + 3) 5 dx = (5 x + 3) 5 (5 x + 3)dx = (5 x + 3) 5 d (5 x + 3)
5 5

§Æt u = 5x + 3 ta ®−îc:
6
1 1 5 1 u
∫ (5x + 3) dx = 5 ∫ (5x + 3) d (5 x + 3) = 5 ∫ u du = 5 . 6
5 5
+C

21
1 (5 x + 3) (5 x + 3) + C
6 6
= . +C =
5 6 30
c) Bµi tËp tù gi¶i:
T×m c¸c nguyªn hµm sau
a, ∫ (3x + 2) dx
4
b, ∫ (4 x − 3) dx
3

c, ∫ (6 x − 1) dx
6
d, ∫ (2 − 5x ) dx
5

2.2. D¹ng : I = ∫ n
ax + b dx

a) C¸ch gi¶i:
§Æt u = ax+b
b) VÝ dô : T×m I = ∫ 3 2 x + 3dx
1 1
1 1
Ta cã : 3 2 x + 3dx = (2 x + 3) 3 (2 x + 3)′dx = (2 x + 3) 3 d (2 x + 3)
2 2

§Æt: u = 2x + 3 ta ®−îc:
1 1 4 4
1 1 1 3 3
∫ 3
2 x + 3dx = ∫ (2 x + 3) 3 d (2 x + 3) = ∫ u 3 du = . u 3 + C = (2 x + 3) 3 + C
2 2 2 4 8
c) Bµi tËp tù gi¶i:
T×m c¸c nguyªn hµm sau
a, ∫ 3
x − 5dx b, ∫ 5
3 x + 4dx

c, ∫ 4
2 x − 3dx d, ∫ 3
3 − 5 x dx

2.3. D¹ng : I = ∫ e ax+b dx

a) C¸ch gi¶i :
§Æt u = ax + b
b) VÝ dô : T×m : ∫ e 2 x+5 dx

Ta cã : e 2 x +5 dx = e 2 x +5 ( 2 x + 5)′dx = e 2 x +5 d ( 2 x + 5)

§Æt u = 2x + 5 ta ®−îc
1 1 u 1 1
∫e dx = ∫2e ∫
2 x +5 2 x +5
d (2 x + 5) = e du = e u + C = e 2 x+5 + C
2 2 2

c) Bµi tËp tù gi¶i:


T×m c¸c nguyªn hµm sau
a, ∫ e 2 x−3 dx b, ∫ e 3 x+7 dx c, ∫ e 5−3 x dx c, ∫ e 5 x+6 dx

22
1
2.4. D¹ng : I= ∫ (ax + b) n
dx

a) C¸ch gi¶i
§Æt u = ax + b
1
b) VÝ dô : T×m I = ∫ (3x + 2) 5
dx

1 1 1 1 1
Ta cã: 5
dx = 5
(3 x + 2)′dx = d (3 x + 2)
(3 x + 2) 3 (3 x + 2) 3 (3 x + 2) 5

§Æt u = 3x + 2 ta ®−îc :
1 1 1 1 1 u −4 1 1
∫ 3 (3x + 2) 5 d (3 x + 2) = ∫
3 u 5
du = .
3 −4
+C = −
12u 4
+C = −
12(3 x + 2) 4
+C

c) Bµi tËp tù gi¶i:


T×m c¸c nguyªn hµm sau
1 1
a, ∫ ( 4 x − 2) 5
dx b, ∫ (7 x + 3) 3
dx

1 1
c, ∫ (3x − 2) 5
dx d, ∫ (5 x + 2) 7
dx

3. D¹ng 3: ¸p dông nguyªn hµm tõng phÇn


C«ng thøc nguyªn hµm tõng phÇn ∫ udv = uv − ∫ vdu

3.1 D¹ng 1: I = ∫ (ax + b)e x dx .

Ph−¬ng ph¸p: §Æt u = ax + b, dv = e x dx


3.2 D¹ng 2: I = ∫ (ax + b) cos xdx

Ph−¬ng ph¸p: §Æt u = ax + b, dv = cos xdx


3.3 D¹ng 3: I = ∫ (ax + b) sixdx

Ph−¬ng ph¸p: §Æt u = ax + b, dv = sin xdx


3.4 D¹ng 4: I = ∫ (ax + b) ln xdx

Ph−¬ng ph¸p: §Æt u = ax + b, dv = ln xdx


3.5 D¹ng 5: I = ∫ x n ln xdx

Ph−¬ng ph¸p:§Æt u = x n , dv = ln xdx


3.6 VÝ dô T×m: I = ∫ (2 x + 1)e x dx

23
Gi¶i
u = 2 x + 1 du = 2dx
§Æt  x
⇒  x
e dx = dv v = e
⇒ I = ∫ (2 x + 1)e x dx = (2x + 1) - 2 ∫ e x dx =2x + 1 – 2 e x + C

3.7 Bµi tËp tù gi¶i


T×m c¸c nguyªn hµm sau

a, ∫ (2 − x )sin xdx ; b, ∫ (3 − 2 x) cos xdx ;

c, ∫ (3x + 5) ln xdx ; d, ∫ (3 − 5 x)e x dx ; e, ∫ x 3 ln xdx

B. tÝch ph©n
I. kiÕn thøc c¬ b¶n
1. C«ng thøc tÝnh tÝch ph©n:

b b

∫ f ( x)dx = F ( x)
a
= F (b) − F (a ) trong ®ã F(x) lµ mét nguyªn hµm cña hµm f(x) trªn [a; b]
a

2. TÝnh chÊt cña tÝch ph©n:


a
1. ∫ f ( x)dx = 0
a
a b
2. ∫ f ( x)dx = − ∫ f ( x)dx
b a
b b
3. ∫ af ( x)dx = a ∫ f ( x)dx ;a∈R
a a
b b b
4. ∫ [ f ( x) ± g ( x)] dx = ∫ f ( x)dx ± ∫ g ( x)dx
a a a
c b c
5. ∫ f ( x)dx = ∫ f ( x)dx + ∫ f ( x)dx
a a b

24
II. Ph−¬ng ph¸p tÝch ph©n:
1. Ph−¬ng ph¸p ®æi biÕn sè
Qui t¾c:

1. §Æt t=v(x), v(x) lµ mét hµm sè cã ®¹o hµm liªn tôc


2. BiÓu thÞ f(x)dx theo t vµ dt. Gi¶ sö f(x)dx=g(t)dt
3. T×m mét nguyªn hµm G(t) cña g(t)
v (b ) v (b )

4. TÝnh ∫
v(a)
g (t )dt = G (t )
v(a)

b v (b )

5. KÕt luËn ∫ f ( x)dx = G (t )


a
.
v(a )

π
4
1.1. VÝ dô tÝnh tÝch ph©n sau: 1) I = ∫ tgxdx
0

1.2. H−íng dÉn


π π
4 4
sin x
Ta cã I = ∫ tgxdx = ∫ cos x dx
0 0

§Æt t = cosx ⇒ dt = - sinxdx ⇒ sinxdx = - dt


2
CËn ®æi: x = 0 ⇒ t = 1; x= π ⇒ t=
4 2
2
2
1 2 2
Khi ®ã I = − ∫ t dt = − ln t 2 = − ln
1 1 2

1.3. Bµi tËp tËp tù gi¶i


TÝnh c¸c tÝch ph©n sau
π π
e
2
sin x 1 + ln x 2
1) I = ∫ dx 2) I = ∫ dx 3) I = ∫ sin
3
x cos xdx ;
0 1 + 3 cos x x 0
1

π π
2 6
4) I = ∫ e sin x cos xdx ; 5) I = ∫ 1 + 4 sin x cos xdx
0 0

2. TÝch ph©n tõng phÇn


2.1 C«ng thøc tÝch ph©n tõng phÇn

b b b

∫ udv = u.v
a
− ∫ vdu
a
a

25
π
6
2.2. VÝ dô TÝnh tÝch ph©n I = ∫ (2 − x )sin 3 xdx
0

Gi¶i
 u = 2 − x ⇒ du = −dx
§Æt sin 3 xdx = dv ⇒ v = − 1 cos 3 x
 3
π

1 π 6
π
I =− (2 − x )cos 3x 6 − 1 ∫ cos 3xdx = − 23 − 91 sin 3x 6 = − 79
3 0 30 0

2.3. Bµi tËp tù gi¶i


TÝnh c¸c tÝch ph©n sau
π
π
2 e 2
1/ a, I = ∫ x sin xdx ; 2
b, I = ∫ ln xdx ; c, I = ∫e
x
sin xdx ;
0 1 0

2 2
x2 − 2x 2 x + 6 x 2 − 4 x3
2/ a, ∫ dx b, ∫1 −2 x3 dx
1
x
π
8 3
dx 2 − cos3 x
c, ∫
1
3
x2
d, ∫ cos 2 x dx
π
6

1 6 1 1
2 x2
3/ a, ∫ x ( x 2 + 1) dx b, ∫ dx c, ∫x e
2 − x3
dx
0 0 1 + x3 −1

π
3 π 1
sin x
d, ∫ 3 dx e, ∫ x.sin xdx f, ∫ x.e2 x dx
0
cos x 0 0

π
2 π 2
g, ∫ ( 2 x + 1) ln xdx h, ∫ x 2 .cos xdx i, ∫ sin 3 x.cos3 xdx
1 0 0

π π
4 e2 2
etgx dx
j, ∫ 2 dx k, ∫ l, ∫ sin 5 xdx
0
cos x e
x.ln x 0

π
1 e 2
m, ∫ ( x + 3)e dx
x
n, ∫ e x .cos 2 xdx p, ∫ e x .cos xdx
−1 0 0

π π
1
2
sin 2 x 2
3x 2
4/ a, I = ∫ dx b, J = ∫ ( x + sin 2 x ) cos xdx d, M = ∫ dx
0
4 − cos 2 x 0 0
x3 + 1

26
Chñ ®Ò H×nh häc kh«ng gian

1. D¹ng 1 : ThÓ tÝch khèi ®a diÖn


C«ng thøc tÝnh thÓ tÝch cña khèi chãp, khèi l¨ng trô, khèi hép ch÷ nhËt

1
VKC = Bh; VKLT = Bh; VKHCN = a.b.c
3
B = S day′ ; h = Chieˆ`u cao.

1.1. VÝ dô
Cho h×nh chãp S.ABCD cã ®¸y ABCD lµ h×nh vu«ng c¹nh a, c¹nh bªn SA vu«ng gãc víi ®¸y,
c¹nh bªn SB b»ng a 3 . TÝnh thÓ tÝch cña khèi chãp S.ABCD theo a .
1.2. H−íng dÉn

Theo gi¶ thuyÕt ta cã :SA lµ chiÒu cao cña h×nh chãp S

SA = 3a 2 − a 2 = a 2

DiÖn tÝch ®¸y ABCD lµ S = a.a = a 2


A D
1 2 3
VËy thÓ tÝch cña khèi chãp lµ : V = a 2 .a 2 = a
3 3 a 3

a
B C

1.3. Bµi tËp tù gi¶i


1. Cho h×nh chãp tø diÖn ®Òu S.ABCD cã AB = a và SA = b . TÝnh thÓ tÝch cña khèi chãp
S.ABCD theo a và b.
2. Cho h×nh chãp tø diÖn ®Òu S.ABCD cã AB = a và gãc SAC b»ng 450 .TÝnh thÓ tÝch cña khèi
chãp S.ABCD theo a và b
3. Cho h×nh chãp tam gi¸c S.ABC cã ®¸y ABC lµ tam gi¸c vu«ng t¹i ®Ønh B, c¹nh bªn SA vu«ng
gãc víi ®¸y. BiÕt SA = AB = BC = a. TÝnh thÓ tÝch cña khèi chãp S.ABC theo a.
4. Cho h×nh chãp tø diÖn ®Òu S.ABCD cã AB = a và gãc gi÷a mÆt bªn vµ mÆt ®¸y b»ng 600 .TÝnh
thÓ tÝch cña khèi chãp S.ABCD theo a
5. Cho khèi hép ch÷ nhËt ABCDA’B’C’D’ cã thÓ tÝch V. TÝnh thÓ tÝch cña khãi tø diÖn C’ABC theo
V.
6. Trªn c¹nh CD cña tø diÖn ABCD lÊy ®iÓm M sao cho CD = 3CM.TÝnh tØ sè thÓ tÝch cña hai tø
diÖn ABMD vµ ABMC.
27
2. D¹ng 2: DiÖn tÝch xung quanh, thÓ tÝch cña khèi cÇu, khèi nãn, khèi trô

KiÒn thøc c¬ b¶n


4
1. Cho khèi cÇu S(I,R) khi ®ã S = 4πR 2 , V = πR 3 víi S lµ diÖn tÝch mÆt cÇu, V lµ thÓ tÝch
3
khèi cÇu
2. Cho h×nh trô (T) cã chiÒu cao h, b¸n kÝnh ®¸y R khi ®ã : S xq = 2πRh,V = πR 2 h víi S lµ
diÖn tÝch xung quanh cña h×nh trô, V lµ thÓ tÝch cña khèi trô
3. Cho h×nh nãn (N) cã ®−êng cao h, ®−êng sinh l, b¸n kÝnh ®¸y R khi
1
®ã S xq = πRh, V = πR 2 h víi S lµ diÖn tÝch xung quanh cña h×nh nãn, V lµ thÓ tÝch cña khèi nãn
3

3. Ph−¬ng ph¸p x¸c ®Þnh t©m cña mÆt cÇu ngo¹i tiÕp h×nh chãp

B−íc 1 : T×m t©m ®−êng trßn ngo¹i tiÕp ®a gi¸c ®¸y


B−íc 2 : Dùng ®−êng th¼ng ®i qua t©m vµ vu«ng gãc víi mÆt ph¼ng ®¸y( ®−êng th¼ng nµy
gäi lµ trôc ®−êng trßn)
B−íc 3 : Dùng mÆt ph¼ng trung trùc cña mét c¹nh bªn bÊt kú
B−íc 4 : Dùng giao cña mÆt trung trùc vµ trôc ®−êng trßn (®iÓm nµy lµ t©m cña mÆt cÇu)

4. Ph−¬ng ph¸p t×m b¸n kÝnh cña t©m mÆt cÇu :


Dùa vµo c¸c tam gi¸c vu«ng ®ång d¹ng hoÆc c¸c tam gi¸c vu«ng, ®Òu
5. VÝ dô: Cho h×nh chãp tø diÖn ®Òu S.ABCD cã AB = a và gãc gi÷a mÆt bªn vµ mÆt ®¸y b»ng
600 .TÝnh thÓ tÝch cña khèi cÇu ngo¹i tiÕp h×nh chãp S.ABCD theo a
5.1. H−íng dÉn
DÒ thÊy giao ®iÓm O cña AC vµ Db lµ t©m cña ®¸y ABCD, V× h×nh S

chãp S.ABCD ®Òu nªn SO vu«ng gãc víi ®¸y P


Gäi (Q) lµ mÆt ph¼ng trung trùc cña SA, (Q) c¾t SA t¹i P, SO
A D
t¹i I. khi ®ã I lµ t©m cña mÆt cÇu ngo¹i tiÕp khèi chãp I
N
B¸n kÝnh cña mÆt cµu lµ IS.TÝnh IS M
O
Theo gi¶ thuyÕt ta cã gãc SMN b»ng 600 nªn SO = MO. tan 600
B C
a 3 a
hay MO =
2
5
SA = a ( Tam gi¸c SAO vu«ng t¹i O)
2
28
a 5 a 5
.
SA SO SA.SP 2 4 = 5a 3
DÔ thÊy ∆SAO ~ SIP nªn ta cã : = ⇒ SI = =
SI SP SO a 3 12
2
2 3
 5a 3  πa 2 4 3 4  5a 3 
VËy S = 4πR = 4.π .
2
 = 25 , V = πR = π  
12  12 3 3 12 
   

5.2. Bµi tËp tù gi¶i:


1. Mét mÆt cÇu ®i qua t¸m ®Ønh cña mét h×nh lËp ph−¬ng c¹nh a.
a. TÝnh b¸n kÝnh cña mÆt cÇu theo a
b. TÝnh diÖn tÝch vµ thÓ tÝch cña h×nh cÇu
2. Cho h×nh chãp S.ABCD cã ®¸y ABCD lµ h×nh vu«ng c¹nh a, c¹nh bªn SA vu«ng gãc víi ®¸y,
c¹nh bªn SB b»ng a 3 . TÝnh thÓ tÝch cña khèi cÇu ngo¹i tiÕp h×nh chãp S.ABCD theo a .
3. Cho h×nh chãp tam gi¸c S.ABC cã ®¸y ABC lµ tam gi¸c vu«ng t¹i ®Ønh B, c¹nh bªn SA vu«ng
gãc víi ®¸y. BiÕt SA = AB = BC = a. TÝnh thÓ tÝch cña khèi cÇu ngo¹i tiÕp h×nh chãp S.ABC theo
a.
4. Cho h×nh chãp tam gi¸c ®Òu S.ABC cã AB = a và gãc gi÷a c¹nh bªn vµ mÆt ®¸y b»ng 600 .
a/ TÝnh thÓ tÝch cña khèi cÇu ngo¹i tiÕp h×nh chãp S.ABC theo a
b/ TÝnh diÖn tÝch cña khãi cÇu
5/ Cho h×nh trô cã b¸n kÝnh ®¸y R vµ diÖn tÝch xung quanh b»ng S xq = 4πR 2 .TÝnh thÓ tÝch cña
khèi trô.
6/ Cho h×nh trô cã b¸n kÝnh ®¸y R vµ thÓ tÝch V = 3πR 3 . TÝnh diÖn tÝch xung quanh cña khèi trô.
7/ Cho h×nh trô cã thiÕt diÖn ®i qua trôc lµ tam gi¸c vu«ng c¹nh 4a. TÝnh diÖn tÝch xung quanh,
thÓ tÝch cña khèi trô.
8/ Cho h×nh trô cã chiÒu cao h = 2a. vµ diÖn tÝch xung quanh S xq = 4πa 2 . TÝnh thÓ tÝch cña khèi
trô, diÖn tÝch toµn phÇn cña khèi trô.
9/ TÝnh thÓ tÝch vµ diÖn tÝch xung quanh cña h×nh trô cã b¸n kÝnh ®¸y R, gäi A vµ B lµ hai ®iÓm
thuéc hai ®¸y sao cho AB = 3R vµ gãc gi÷a AB vµ OO’ b»ng 300 .( O, O’ lÇn l−ît lµ t©m cña c¸c
®¸y) .
HD : Bµi tËp tõ 6-10 thuéc lo¹i h×nh trô, khèi trô
10/ Cho h×nh nãn cã chiÒu cao h , b¸n kÝnh ®¸y R. TÝnh diÖn tÝch xung quanh, thÓ tÝch cña khèi
nãn
11/ Cho h×nh nãn cã chiÒu cao h, ®−êng sinh l. TÝnh diÖn tÝch xung quanh, thÓ tÝch cña khèi nãn.
12/ Cho h×nh nãn cã chiÒu cao h, gãc ë ®Ønh b»ng 1200 . TÝnh diÖn tÝch xung quanh, thÓ tÝch cña
khèi nãn.

29
13/ Cho h×nh nãn cã chiÒu cao h, gãc hîp bëi ®−êng sinh vµ ®¸y b»ng 300 . TÝnh diÖn tÝch xung
quanh, thÓ tÝch cña khèi nãn.
14/ Cho h×nh nãn cã chiÒu cao h, gãc hîp bëi ®−êng sinh vµ ®−êng cao 300 . TÝnh diÖn tÝch xung
quanh, thÓ tÝch cña khèi nãn.
15/ Cho h×nh nãn cã thÓ tÝch b»ng V vµ chiÒu cao h. TÝnh diÖn tÝch xung quanh cña khèi nãn
theo V vµ h.
Chó ý : Khi tÝnh thÓ tÝch hoÆc diÖn tÝch xung quanh cña khèi nãn, h×nh nãn. Ta cÇn x¸c ®Þnh ®−îc
R, h, l.
*NÕu biÕt h, l th× t×m R b»ng c«ng thøc R = l 2 − h 2

*NÕu biÕt h, R th× t×m l b»ng c«ng thøc l = R 2 + h 2


*NÕu biÕt R, l th× t×m h b»ng c«ng thøc h = l 2 − R 2
R
* NÕu biÕt R, vµ gãc ë ®Ønh 2α th× : h = R cot α , l =
sin α
* NÕu biÕt l, vµ gãc ë ®Ønh 2α th× : h = l. cos α , R = l. sin α
h
* NÕu biÕt h, vµ gãc ë ®Ønh 2α th× : l = , R = coh. tan α
cos α

* NÕu biÕt gãc t¹o bëi ®−êng sinh vµ c¸c ®¸y vµ mét trong ba yÕu tè R, h, l th× lµm t−¬ng tù nh−
trªn

30
PhÇn riªng Theo ch−¬ng tr×nh chuÈn

CHỦ ĐỀ PHÖÔNG PHAÙP TOÏA ÑOÄ TRONG KHOÂNG GIAN

I. TOÏA ÑOÄ CUÛA VECTÔ VAØ CUÛA ÑIEÅM


1.Toïa ñoä cuûa vectô

Ñònh nghóa: Trong không gian với hệ tọa độ Oxyz cho vectơ u tuøy yù ,do
→ → → → → →
i , j , k khoâng ñoàng phaúng neân toàn taïi boä ba soá thöïc (x ; y ; z) sao u = x i + y j +

zk
→ →
Boä ba soá (x ; y ; z) goïi laø toïa ñoä cuûa vectô u , kí hieäu: u = ( x ; y ; z )
→ → → → →
Vaäy u=(x;y;z)⇔ u =x i +y j +zk
→ →
Caùc tính chaát: u = ( x ; y ; z ) , v = ( x’ ; y’ ; z’ )
→ →
• u + v = ( x + x’ ; y + y’; z + z’ )
→ →
• u - v = ( x – x’ ; y – y’; z – z’ )

• k u = ( kx ; ky ; kz )
x = x'
→ → 
• u = v ⇔ y = y'
z = z '

2. Toïa ñoä cuûa ñieåm :


Ñònh nghóa: Trong kg(Oxyz ) cho ñieåm M tuøy yù. Toïa ñoä cuûa vectô OM ñöôïc goïi
laø toïa cuûa ñieåm M .
−→
Vaäy neáu OM = (x ; y ; z) thì boä ba soá (x ; y ; z) laø toïa ñoä cuûa ñieåm M ,

Ta vieát : M ( x ; y ; z )
→ → →
−→
M(x;y;z) ⇔ OM = x i + y j + z k

Caùc tính chaát : A ( xA ; yA ; zA ) , B ( xB ; yB ; zB ) ta coù ;


• AB = ( xB – xA ; yB – yA ; zB – zA )

• AB = (xB − xA )2 + ( yB − yA )2 + (zB − z A )2

31
 x A − kx B
xM = 1 − k


−→ −→
 y − ky B
MA = k MB , (k ≠ 1) ⇔  y M = A
 1− k
 z A − kz B
z M = 1 − k

 x A + xB
xM = 2

• M laø trung ñieåm cuûa ñoaïn AB ⇔  y M = y A + y B
 2
 z A + zB
z M =
 2

 1
 xG = 4 ( x A + x B + xC + x D )

 1
• G(xG;yG; zG) laø troïng taâm töù dieän ABCD ⇔  y G = 4 ( y A + y B + yC + y D )

 1
zG = 4 ( z A + z B + zC + z D )

3 .Bieåu thöùc toïa ñoä cuûa tích voâ höôùng cuûa hai vectô :
→ →
Cho hai vectô a = ( x1; y1 ; z1 ) , b = ( x2 ; y2 ; z2 ) ta coù :
→ →
* a . b = x1x2 + y1y2 + z1z2
→ →
* a ⊥ b ⇔ x1x2 + y1y2 + z1z2 = 0

*| a |= x12 + y12 + z12
x1x2 + y1 y2 + z1z2
* cos ϕ =
x12 + y12 + z12 . x22 + y22 + z22
→ →
* a vaø b cuøng phöông vôùi nhau ⇔ x1: y1: z1= x2 : y2: z2
4 . Tích coù höôùng cuûa hai vectô:
→ →
a. Ñònh nghóa : Cho hai vectô a = ( x1; y1 ; z1 ) , b = ( x2 ; y2 ; z2 ). Tích coù höôùng
→ → → →
cuûa hai vectô a vaø b laø moät vectô kí hieäu laø [ a , b ] vaø
→ →  y z1 z1 x1 x1 y 
[ a , b ] =  
1 1
; ; 
 y 2 z 2 z 2 x 2 x 2 y 2 
→ → →
Ví du 1ï: Cho ba vectô a = ( 2;1 ; 0 ), b = ( 1; -1; 2) , c = (2 ; 2; -1 ). Tìm toïa ñoä cuûa
→ → → →
vectô : u = 4 a - 2 b + 3 c .
32
Giaûi:

Ta coù: 4 a (4.2; 4.1; 4.0) = (8; 4; 0)

2 b (2.1; 2.(-1); 2.2) = (2; -2; 4)

Töông töï : 3 c (6; 6; -3)
→ → → →
⇒ u = 4 a - 2 b + 3 c = (8 – 2 + 6; 4 + 2 + 6; 0 – 4 - 3) = (12; 12; -7 )

Baøi taäp
1. Vieát toïa ñoä cuûa caùc vectô sau :
→ → → → → → → → → → → →
a = −2 i + 3 j − 4 k , b = 2 j− i , c = 3k , d = −k− 2 i
→ → →
2. Cho ba vectô a = ( 2;-5 ; 3 ), b = ( 0; -2; 1) , c = (-1 ; 6; 2 ).Tìm toïa ñoä cuûa vectô
→ → → → 1→ → →
u = 2 a - b . v = a+ 3 b+ c
2
→ → →
3. Cho ba vectô a = ( 0;-2 ; 4 ), b = ( 1; 3; -1) , c = (2 ; 0; 5 ).Tìm toïa ñoä cuûa :
→ → 1→ →
a) Vectô d = 4 a − b + 3 c.
3
→ → → →
b) Vectô x bieát x + 2 a = − a .
→ → → →
c) Vectô u bieát 2 a + u = 5 b

d) Tìm  a . b . c , e)  b . c . a g )  a , b . c


→ → → → → → → → →
,
     

4. Cho 3 ñieåm A ( 3;-4;7 ),B( -5; 3; -2 ) ,C(1; 2; -3 ).


a. Xaùc ñònh ñieåm D sao cho töù giaùc ABCD laø hình bình haønh .
b. Tìm toïa ñoä giao ñieåm cuûa hai ñöôøng cheùo.
5. Trong khoâng gian vôùi heä toïa ñoä Oxyz cho ba ñieåm A(3;4;-1) , B(2;0;3),C(-3;5;4)
a. Tìm ñoä daøi caùc caïnh cuûa tm giaùc ABC.
b. Tính cosin caùc goùc A,B,C .
6. Trong heä toïa ñoä Oxyz cho ba ñieåm A(1;2 ; -3) , B(3 ; 2 ; 0) , C ( -4; 2 ; 5).
a) Chöùng minh A , B ,C laø ba ñænh cuûa moät tam giaùc .
b) Tìm toïa ñoä ñieåm D sao cho töù giaùc ABCD laø hình bình haønh .
7. Trong khoâng gian vôùi heä toïa ñoä Oxyz , cho ba ñieåm A(1; -1;-3) ,B(2 ;1 ; -2) , C(-5 ; 2 ;
-6) .
a) Chöùng minh A, B , C laø ba ñænh cuûa tam giaùc .

33
b) Tính ñoä daøi phaân giaùc ngoaøi goùc A cuûa tam giaùc ABC.
c) Tìm toïa ñoä tröïc taâm cuûa tam giaùc ABC.
II . PHÖÔNG TRÌNH TOÅNG QUAÙT CUÛA MAËT PHAÚNG
A. Lí thuyeát caàn nhôù :
1. Ñònh nghóa :
→ →
• Vectô n ≠ 0 ñöôïc goïi laø vectô phaùp tuyeán cuûa maët phaúng (α ) neáu noù naèm
treân ñöôøng thaúng vuoâng goùc vôùi ( α ).

Kí hieäu : n ⊥ ( α )
→ → →
• Trong khoâng gian vôùi heä toïa ñoä Oxyz neáu hai vectô a , b ≠ 0 , khoâng cuøng
phöông vaø caùc ñöôøng thaúng chöùa chuùng song song hoaëc naèm trong (α ) ñöôïc
goïi laø caëp vectô chæ phöông cuûa maët phaúng ( α ).
Chuù yù :
→ → → → →
Neáu ( α ) coù caëp vectô chæ phöông a , b thì (α ) coù moät vectô phaùp tuyeán n = [ a , b ]
2.Phöông trình maët phaúng:

M aët phaúng ( α ) qua M0( x0 ;y0 ; z0 ) coù vtpt n = ( A; B; C ) coù phöông trình laø :

A ( x – x0 ) + B (y – y0) + C ( z – z0 ) = 0

Phöông phaùp chung laäp phöông trình cuûa maët phaúng :


Ñeå laäp phöông trình cuûa moät maët phaúng ta caàn tìm moät ñieåm thuoäc maët phaúng
vaø vtpt cuûa noù hay tìm caëp vtcp cuûa noù

V í dụ
(α) laø maët phaúng trung tröïc cuûa ñoaïn AB Vieát phöông trình toång quaùt cuûa maët
phaúng ( α ) trong caùc tröôøng hôïp sau:
(α) ñi qua M (3; 2; -5 ) vaø vuoâng goùc vôùi truïc Oz .vôùi A( 3; -5; 4 ), B( 1 ; 3; -2 ).
Giaûi:

+ Maët phaúng(α) ñi qua M (3; 2; -5 ) vaø vuoâng goùc vôùi truïc Oz nhaän k (0;0;1) laøm
vectô phaùp tuyeán coù phöông trình toång quaùt laø:
0(x - 3) + 0(y -2) + 1(z + 5) = 0 ⇔ z + 5 = 0
+ Goïi I laø trung ñieåm cuûa AB ta coù I(2;-1;1)

34
Ta coù AB(−2;8;−6) . Maët phaúng trung tröïc cuûa ñoaûn AB ñi qua I vaø nhaän AB(−2;8;−6)
laøm vectô phaùp tuyeán coù phöông trình toång quaùt laø:
-2(x - 2) + 8(y + 1) -6(z - 1) = 0
⇔ -2x + 8y -6z + 18 = 0

Baøi taäp
1.Vieát phöông trình toång quaùt cuûa maët phaúng ( α ) trong caùc tröôønghôïp sau:
a.(α) ñi qua M(2 ; -1 ; -3) vaø vuoâng goùc vôùi truïcc Ox .
b.(α) laø maët trung tröïc cuûa ñoaïn AB vôùi A(1; 3; 2 ), B(-1 ; 1; 0 ).
c. (α) qua I(-1; 2;4 ) vaø song song vôùi maët phaúng 2x – 3y + 5z – 1 = 0.
2.Vieát phöông trình maët phaúng (α) trong caùc tröôøng hôïp sau:
a. (α) ñi qua hai ñieåm M( 1; -1; 2 ) , vaø vuoâng goùc vôùi truïc Oz .
b. (α) ñi qua ba ñieåm A(1; 6; 2 ), B( 5; 0; 4), C( 4; 0; 6 ) .
3.Vieát phöông trình maët phaúng :
a. Ñi qua A( 1 ; 0 ; 2) vaø song song vôùi maët phaúng xOy.
b. Ñi qua M(2 ;-1 ; -3) vaø vuoâng goùc vôùi truïc Ox .
c. Ñi qua I( -1 ; 2 ; 4) vaø song song vôùi maët phaúng (P): 2x – 3y + 5z – 1 = 0 .
d. (α ) laø maët trung töïc cuûa ñoaïn AB vôùi A(1 ; 2 ; 3) , B(-1 ; 1 ; 0).
e. (β ) ñi qua ba ñieåm A(-1 ; 2 ; 3) ,B(2 ; -4 ; 3) , C(4 ; 5 ; 6).
4.Vieát phöông trình maët phaúng :
a. Ñi qua hai ñieåm A(1 ;1 ;0) ,B(-1 ; 2 ; 7) vaø vuoâng goùc vôùi maët phaúng (α) :2x–3y+z–
7 = 0.
b. Ñi qua M(0 ;2; -1) , song song vôùi truïc Ox vaø vuoâng goùc vôùi maët phaúng (β) x – y
+z = 0 .
c. Ñi qua N(-3;0;1) vaø vuoâng goùc vôùi hai maët phaúng (P):2x–3y+z –2 = 0 ;(Q):x + 5y–
2z = 0
5.Cho töù dieän ABCD coù A(5 ; 1 ; 3) ,B(1 ; 6 ; 2) , C(5 ; 0 ; 4) ,D(4 ; 0 ;6) .
a. Vieát phöông trình maët phaúng (BCD).
b. Vieát phöông trình maët phaúng ñi qua AB vaø song song vôùi CD .
c. Goïi G laø troïng taâm cuûa tam giaùc BCD . Vieát phöông trình maët phaúng ñi qua G vaø
song song vôùi maët phaúng (ABC ) .

35
III. ÑÖÔØNG THAÚNG
A. Lí thuyeát caàn nhôù
→ →
Vectô u ≠ 0 naèm treân ñöôøng thaúng song song hoaëc truøng vôùi ñöôõng thaúng (d)
goïi laø vectô chæ phöông cuûa ñöôøng thaúng (d).

Ñöôøng thaúng (d) ñi qua ñieåm M0( x0; y0 ; z0 ) coù vectô chæ phöông u = ( a; b; c)
 x = x 0 + at

coù phöông trình tham soá laø :  y = y 0 + bt t∈R
 z = z 0 + ct

x − x0 y − y0 z − z0
Phöông trình chính taéc : = = .
a b c

B.Phöông phaùp chung ñeå laäp phöông trình cuûa ñöôøng thaúng:
Ñeå laäp phöông trình cuûa moät ñöôøng thaúng ta söû duïng moät trong hai caùch sau:
• Tìm vectô chæ phöông cuûa ñöôøng thaúng vaø moät ñieåm thuoäc ñöôøng thaúng.
Chuù yù :+ Hai ñöôøng thaúng song song coù cuøng vectô chæ phöông.
+Neáu ñöôøng thaúng vuoâng goùc vôùi maët phaúng thì noù nhaän vtpt cuûa maët phaúng
laøm vtcp.
Caùch giaûi:
• Vieát phöông trình maët phaúng (α) qua M vaø vuoâng goùc vôùi ( ∆ ).
• Vieát phöông trình maët phaúng (β) qua M vaø ( ∆ ).
Chuù yù : Neáu (∆) ⊥ mp(α) thì (∆) nhaän VTPT cuûa (α) laøm VTCP
Ví dụ 3: Vieát phöông trình tham soá vaø chính taéc cuûa ñöôøng thaúng ( ∆ ):
a. Qua hai ñieåm M( 2; -3; 5), N( 1; -2; 3).
b. Qua A(1; -1; 3) vaø song song vôùi BC trong ñoù B(1; 2; 0 ),C(-1; 1; 2)
c. Qua D(3; 1; -2) vaø vuoâng goùc vôùi maët phaúng 3x + 4y – 2z +5 = 0
Giaûi:
a. Ñöôøng thaúng ( ∆ ) ñi qua ñieåm M( 2; -3; 5) nhaän vectô MN (−1;1;−2) laøm vectô chæ
phöông coù phöông trình :
x = 2 − t

+ Phöông trình tham soá:  y = −3 + t , t ∈ R
 z = 5 − 2t

x −2 y +3 z −5
+ Phöông trình chính taéc: = =
−1 1 −2
b. . Ñöôøng thaúng ( ∆ ) ñi qua ñieåm A( 1; -1; 3) song song vôùi BC nhaän vectô
BC (−2;−1;2) laøm vectô chæ phöông coù phöông trình :

36
 x = 1 − 2t

+ Phöông trình tham soá:  y = −1 − t , t ∈ R
 z = 3 + 2t

x −1 y +1 z − 3
+ Phöông trình chính taéc: = =
−2 −1 2
c. Ñöôøng thaúng ( ∆ ) ñi qua ñieåm D( 3; 1; -2) vaø vuoâng goùc vôùi maët phaúng

3x + 4y – 2z +5 = 0 nhaän vectô phaùp tuyeán cuûa maët phaúng n (3;4;−2) laøm
vectô chæ phöông coù phöông trình:
 x = 3 + 3t

+ Phöông trình tham soá:  y = 1 + 4t , t ∈ R
 z = −2 − 2t

x − 3 y −1 z + 2
+ Phöông trình chính taéc: = =
3 4 −2
C. Baøi taäp :
1. Cho A(4; -3; 2), B(-2; 1; -4)
a. Viết PT mặt phẳng trung trực của đoạn thẳng AB
b. Viết PT mặt phẳng quá A, B và song song với Ox.
x = 1− t

2. Cho đường thẳng d:  y = −1 + t và (P): x + 2y + z - 5 = 0
 z = 1 + 2t

Viết phương trình hình chiếu vuông góc d của A lên (P).
3. Vieát phöông trình tham soá , phöông trình chính taéc cuûa ñöôøng thaúng ñi qua hai
ñieåm
A(-1 ; 4 ; 3) ,B(2 ; 1 ; 1).
4. Vieát phöông trình chính taéc cuûa ñöôøng thaúng :
 x = 1 + 3t

a) Ñi qua ñieåm M( 1 ; - 2 ; 3) vaø song song vôùi ñöôøng thaúng :  y = −3 − t
 z = 4t

b) Ñi qua ñieåm N( 2 ; 3 ; - 4) vaø vöoâng goùc vôùi maët phaúng x -2y + z – 6 = 0
IV. VÒ TRÍ TÖÔNG ÑOÁI CUÛA CAÙC ÑÖÔØNG THAÚNG VAØ MAËT PHAÚNG.

A. LÍ THUYEÁT :
1/ Vò trí töông ñoái cuûa hai ñöôøng thaúng:
x − x0 y − y 0 z − z 0 x − x0' y − y 0' z − z 0'
Cho hai ñöôøng thaúng : (d) : = = ,( d’ ): = =
a b c a' b' c'

(d) qua M0(x0 ;y0 ;z0) ,coù VTCP u = ( a; b; c)
37

(d’) qua M’0(x’0 ;y’0 ;z’0) ,coù VTCP u ' = ( a’; b’; c’)
→ → − − −→
’ ' '
a. (d) vaø (d ) ñoàng phaúng ⇔ [u , u ]. M 0 M 0 = 0
→ → −−→
' '
b. (d) vaø (d’) caét nhau ⇔ [u , u ]. M 0 M 0 = 0 vaø a:b:c ≠ a’:b’:c’

c. (d)//(d’) ⇔ a:b:c = a’:b’:c’≠ (x’0 – x0 ):(y’0 – y0) :(z’0 – z0)


d. (d) ≡ (d’) ⇔ a:b:c = a’:b’:c’ = (x’0 – x0 ):(y’0 – y0) :(z’0 – z0)
→ → −− −→
' '
e. (d) vaø (d’) cheùo nhau ⇔ [u , u ]. M 0 M 0 ≠ 0

Ví duï: Xeùt vò trí töông ñoái cuûa caùc caëp ñöôøng thaúng sau

x −1 y + 2 z x + y − z + 2 = 0
a. d: = = vaø d’ 
3 1 1 x + 1 = 0
x − y + z − 1 = 0 2 x − y + 1 = 0
b. d:  vaø d’: 
2 x + y + 1 = 0 3 x + y − z + 3 = 0
Giaûi

a. Ñöôøng thaúng (d) ñi qua ñieåm M(1; -2; 0) coù vectô chæ phöông u (3;1;1) . Ñöôøng thaúng

(d’) ñi qua ñieåm M’(-1; 0; 1) coù vectô chæ phöông u , (0;−1;−1) .
 
Tacoù: + [u , u ′] =(0; 3; -3), MM ′ (-4; -1; 0)
 
+ [u , u ′] . MM ′ =0(-4) +3(-1) +(-3)0 = -3 ≠ 0 ⇒ d vaø d’ cheùo nhau

b. Ñöôøng thaúng (d) ñi qua ñieåm M(0; -1; 0) coù vectô chæ phöông u (−1;2;3) . Ñöôøng

thaúng (d’) ñi qua ñieåm M’(0; 1; -4) coù vectô chæ phöông u , (1;2;5) .
 
Tacoù: + [u , u ′] =(4; 8; -4), MM ′ (0; 2; -4)
 
+ [u , u ′] . MM ′ =4.0 + 8.2 -4.(-4) = 0 (1)

+ -1:2:3 ≠ 1:2:5 (2)


Töø (1) vaø (2) ⇒ d vaø d’ caét nhau
Baøi taäp : Xeùt vò trí töông ñoái cuûa caùc caëp ñöôøng thaúng sau
x −1 y − 2 z − 3 x −7 y −6 z −5
a) d: = = vaø d’: = =
9 6 3 6 4 2
x −1 y − 2 z − 3 x−7 y −6 z −5
b) d: = = vaø d’: = =
9 6 3 6 4 2
 x = −2t
x −1 y − 2 z 
c) = = vaø d’ :  y = − 5 + 3 t .
2 −2 1 z = 4

38
2/ Vò trí töông ñoái cuûa hai maët phaúng :
Cho hai mp (α ) , ( β ) lần lượt có phương trình:
(α ) : Ax+By+Cz+D=0
( β ):A’x+B’y+C’z+D=0
a) ( α ) cắt ( β )
⇔ A : B : C ≠ A': B ': C '
A B C D
b) (α ) //( β ) ⇔ = = ≠
A′ B ′ C ′ D ′
A B C D
c) (α ) ≡ ( β ) ⇔ = = =
A' B ' C ' D '

Chú ý: Điều kiện vuông góc giữa 2 mp:


(α ) ⊥ ( β ) ⇔ AA '+ BB '+ CC ' = 0
B. BAØI TAÄP
Baøi1. Xeùt vò trí töông ñoái cuûa caùc caëp maët phaúng sau
a. ( α ): x + 2y – z + 5 = 0 vaø ( β ): 2x + 3y -7z – 4 = 0
b. ( α ) : x – 2y + z + 3 = 0 vaø ( β ): 2x -y + 4z – 2 = 0
c. ( α ): x + y + z - 1 = 0 vaø ( β ): 2x + 2y -2z + 3 = 0
d. ( α ): 3x - 2y –3 z + 5 = 0 vaø ( β ): 9x - 6y -9z – 5 = 0
e.( α ): x - y + 2z -4 = 0 vaø ( β ): 10x - 6y + 20z – 40 = 0

Baøi 2. Xaùc ñònh caùc giaù trò l vaø m ñeå caùc maët phaúng sau ñaây song song vôùi nhau
a. 2x + ly + 2z + 3 = 0 vaø mx+ 2y - 4z + 7 = 0
b. 2x + y + mz - 2 = 0 vaø x + ly + 2z + 8 = 0
Baøi 3. Xeùt vò trí töông ñoái cuûa caùc caëp maët phaúng sau :
a) 2x – 3y + 4z – 5 = 0 vaø 3x – y +z – 1 = 0 .
b) – x +y – z + 4 = 0 vaø 2x – 2y + 2z – 7 = 0.
c) x + y + z – 3 = 0 vaø 2x – 2 y + 2 z – 3 = 0.
d) 3x + 3y – 3z – 12 = 0 vaø 4 x + 4y – 4z – 16 = 0.
Baøi 4 : Cho hai maët phaúng coù phöông trình :(m2–5)x – 2y + mz + m – 5 = 0
vaø x + 2y – 3nz +3 = 0 .
Tìm m , n ñeå hai maët phaúng :
a) Song song vôùi nhau .
b) Truøng nhau .

39
c) Song song vôùi nhau .
d) Truøng nhau .
e) Caét nhau .
Baøi 5 : Cho hai maët phaúng : 3x – (m – 3)y +2z – 5 = 0 vaø (m + 2)x – 2y + mz – 10 = 0
. Tìm m ñeå
a) Hai maët phaúng song song vôùi nhau .
b) Hai maët phaúng truøng nhau .
c) Hai maët phaúng caét nhau .
V. KHOAÛNG CAÙCH, GOÙC
A. LÍ THUYEÁT :
1. Khoaûng caùch töø moät ñieåm tôùi moät maët phaúng

Khoaûng caùch töø ñieåm M0(x0; y0; z0) ñeán maët phaúng (α): Ax + By + Cz + D = 0
Ax0 + By 0 + Cz 0 + D
Kí hieäu: d(M0;(α)) =
A2 + B 2 + C 2

2. Khoaûng caùch töø moät ñieåm tôùi moätñöôøng thaúng


Khoaûng caùch töø ñieåm M ñeán ñöôøng thaúng (∆ ) ñi qua ñieåm M0 vaø coù vectô chæ

phöông u ñöôïc xaùc ñònh theo coâng thöùc:
[M M , u]
0
d(M, ∆) = 
u

3. Khoaûng caùch giöõa hai ñöôøng thaúng cheùo nhau



Cho hai ñöôøng thaúng d1 ñi qua ñieåm M1 coù vecytô chæ phöông u1 vaø ñöôøng thaúng d2

ñi qua ñieåm M2 coù vecytô chæ phöông u 2 . Goïi h laø khoaûng caùch giöõa d1 vaø d2

[u1 , u2 ].M 1 M 2


ta coù: h=
[u1 , u2 ]
B. BAØI TAÄÂP:
1. Tính khoaûng caùch töø caùc ñieåm M1(1;-3;4) , M2( 0;4 ;1) , M3( 2;-1;0 ) ñeán maët
phaúng
(α) : 2x –2y + z – 5 = 0
2. Tính khoaûng caùch töø ñieåm A(1;1;3) tôùi ñöôøng thaúng
x+2 y −1 z +1
∆: = =
1 2 −3
3. Tính khoaûng caùch giöõa hai ñöôøng thaúng cheùo nhau :

40
x + y − z + 5 = 0 x −1 y + 2 z − 3
(∆1):  vaø (∆2): = =
2 x − y + 1 = 0 1 1 −1

4. Tìm treân Oz ñieåm M caùch ñeàu ñieåm A( 2; 3; -1 )vaø maët phaúng:x + 3y +z –17 = 0
5. Trong khoâng gian vôùi heä truïc toïa Oxyz cho ñieåm M(1 ; -2 ; 3) . Tính khoaûng caùch
töø M ñeán :
a) Maët phaúng Oyz .
b) Maët phaúng (P): x – 2y – 2z + 3 = 0.
VI. MAËT CAÀU
A.Lí thuyeát caàn nhôù:
Phöông trình Maët caàu:
a. Maët caàu (S) coù taâm I(a;b;c), baùn kính R coù phöông trình laø:
( x- a )2 + ( y - b )2 + ( z - c )2 = R2

b. Phöông trình : x2+y2+z2 – 2ax – 2by – 2cz + d = 0 ,a2+b2+c2- d > 0


laø phöông trình cuûa maët caàu coù taâm I(a;b;c) , baùn kính R = a 2 + b 2 + c 2 − d
B.Caùc daïng baøi taäp thöôøng gaëp:
1. Tìm taâm vaø baùn kính cuûa maët caàu sau :
a) x2 + y2 + z2 – 8x + 2y +1 = 0
b) x2 + y2 + z2 + 4x + 8y – 2z – 4 = 0
c)3x2 + 3y2 + 3z2 + 6x – 9y + 12z – 4 = 0
2. Vieát phöông trình maët caàu (S) trong caùc tröôøng hôïp sau :
a) (S) coù taâm I ( 1; -2 ; 3 ) vaø ñi qua ñieåm M( 3 ; 2 ; 4 ).
b) (S) coù ñöôøng kính AB vôùi A(1; 4 ; 5), B ( 3; -2; 7 ).
c) (S) coù taâm I( 0 ; 4; 3 ) vaø tieáp xuùc vôùi maët phaúng (α) : 2x + y – 2z + 8 = 0
d) (S) ngoaïi tieáp töù dieän ABCD vôùi A( 3; 2; 6 ), B( 3; -1; 0 ),C( 0; -7; 3 ),D(-2;1; -1).
3. Laäp phöông trình maët caàu (S) ñi qua ba ñieåm A( 1; 2; - 4 ) , B( 1; - 3; 1 )
C( 2; 2; 3 ) vaø coù taâm I naèm treân maët phaúng Oxy.

41
Chñ ®Ò øng dông cña tÝch ph©n
Néi dung träng t©m
* TÝnh diÖn tÝch h×nh ph¼ng
* TÝnh thÓ tÝch khèi trßn xoay
I/ Tãm t¾t lý thuyÕt:
1. C«ng thøc tÝnh diÖn tÝch h×nh ph¼ng: y y
y=f(x)
* Cho miÒn D giíi h¹n bëi c¸c ®−êng: a b
O x
 x = a; x = b (a<b) D
D= D
Ox; y = f ( x) y=f(x)

b
O a b x
Khi ®ã diÖn tÝch miÒn D lµ: S = ∫ f ( x) dx
a

y
* Cho miÒn D' giíi h¹n bëi c¸c ®−êng: y=f1(x)

 x = a; x = b (a<b)
D' =  D
 y = f1 ( x); y = f 2 ( x)
b y=f2(x)
Khi ®ã diÖn tÝch miÒn D' lµ: S ' = ∫ f1 ( x) − f 2 ( x) dx
a
O a b x

2. C«ng thøc tÝnh thÓ tÝch khèi trßn xoay:


* Cho miÒn D giíi h¹n bëi c¸c ®−êng:
 x = a; x = b (a<b) y
D= . Khi cho miÒn D quay quanh trôc
Ox; y = f ( x)
Ox t¹o thµnh khèi trßn xoay; thÓ tÝch khèi trßn xoay ®−îc
b
tÝnh theo c«ng thøc: V = π ∫ f 2 ( x)dx
a
O a b x
II/ Mét sè vÝ dô minh häa:
1. TÝnh diÖn tÝch h×nh ph¼ng.
VÝ dô 1: TÝnh diÖn tÝch h×nh ph¼ng giíi h¹n bëi c¸c ®−êng:
x = 0; x = 1; y = x 2 + 2 x; truc Ox

Gi¶i:
DiÖn tÝch S cña h×nh ph¼ng lµ:
1
1 1 x3 4
S=∫ x + 2 x dx = ∫ ( x + 2 x)dx = ( + x 2 ) =
2 2
0 0 3 0
3

Chó ý: Khi gi¶ thiÕt bµi to¸n ch−a cã cËn cña tÝch ph©n, chóng ta ph¶i ®i t×m cËn cña tÝch ph©n
b»ng c¸ch gi¶i ph−¬ng tr×nh f1 ( x) = f 2 ( x) . NghiÖm t×m ®−îc lµ cËn cña tÝch ph©n.
VÝ dô 2: TÝnh diÖn tÝch h×nh ph¼ng giíi h¹n bëi c¸c ®−êng: y = x 2 − 2 x vµ y = x

42
Gi¶i:
T×m hoµnh ®é giao ®iÓm cña hai ®−êng ta cã: x 2 − 2 x = x ⇔ x = 0; x = 3
VËy diÖn tÝch S cña h×nh ph¼ng lµ:
3
3 3 3 x3 3 x 2 9
S = ∫ ( x − 2 x) − x dx = ∫
2
x − 3x dx = ∫ ( x − 3 x)dx = ( −
2 2
) = .
0 0 0 3 2 0 2

Chó ý: Khi h×nh ph¼ng giíi h¹n bëi nhiÒu ®−êng, ta sö dông c¸ch chia kho¶ng (a;b) thµnh c¸c
kho¶ng nhá vµ sö dông c«ng thøc:
b c b
∫a
f ( x) dx = ∫ f ( x) dx + ∫ f ( x) dx ( a < c < b).
a c

VÝ dô 3: TÝnh diÖn tÝch h×nh ph¼ng giíi h¹n bëi c¸c ®−êng: y = ln x vµ y = 1 .

Gi¶i:
1
XÐt ph−¬ng tr×nh: ln x = 1 ⇔ ln x = ±1 ⇒ x = e; x =
e
Khi ®ã diÖn tÝch S cña h×nh ph¼ng lµ:
e 1 e 1 e
S = ∫1 ln x − 1dx = ∫1 ln x − 1dx + ∫ ln x − 1dx = ∫1 − ln x − 1 dx + ∫ ln x − 1 dx
1 1
e e e

1 e
∫ (ln x + 1)dx + ∫
1 e
S= 1 (ln x − 1)dx = ( x ln x − x + x) 1 + ( x ln x − x − x) 1
1 e
e

(Sö dông kÕt qu¶ ∫ ln xdx = x ln x − x + C ; VD9 SGK Gi¶i tÝch 12 ChuÈn trang 100)

1 1 1
S = x ln x 1 + ( x ln x − 2 x) 1 = 1ln1 − ln + e ln e − 2e + 2 , S = + e − 2
1 e

e e e e

2. TÝnh thÓ tÝch khèi trßn xoay.


VÝ dô 4: TÝnh thÓ tÝch khèi trßn xoay ®−îc t¹o bëi phÐp quay quanh trôc Ox h×nh ph¼ng giíi h¹n
bëi c¸c ®−êng y = x; y = 0; x = 1; x = 2.
Gi¶i:
2
2 x3 8 1 7π
Ta cã: V = π ∫ 2
x dx = π =π( − ) =
1 3 1 3 3 3

VÝ dô 5: Cho h×nh ph¼ng giíi h¹n bëi c¸c ®−êng y = x 2 − 4 x + 4; y = 0; x = 0; x = 3 quay quanh
trôc Ox. TÝnh thÓ tÝch khèi trßn xoay t¹o thµnh.
Gi¶i:
3
3 ( x − 2)5
3 33π
Ta cã: V = π ∫ ( x − 4 x + 4) dx = π ∫ ( x − 2) dx = π
2 2
= 4
0 0 5 0
5

VÝ dô 6: TÝnh thÓ tÝch khèi trßn xoay ®−îc t¹o bëi phÐp quay quanh trôc Ox h×nh ph¼ng giíi h¹n
bëi c¸c ®−êng y = 4 − x 2 ; y = 0 .

43
Gi¶i:
XÐt ph−¬ng tr×nh: 4 − x 2 = 0 ⇔ x = ±2
2 2
VËy V = ∫ (4 − x 2 )2 dx = ∫ (16 − 4 x 2 + x 4 )dx
−2 −2

2
4 x3 x 5 384
V = (16 x − + ) =
3 5 −2 5

III. Bµi tËp tù gi¶i


TÝnh diÖn tÝch h×nh ph¼ng giíi h¹n bëi c¸c ®−êng sau:
1. y = 2 x − x2 ; x + y = 2
2. x + y = 1; x + y = −1; x − y = 1; x − y = −1
1 1
3. y= 2
;y =
1+ x 2
4. y = x 3 − 1 vµ tiÕp tuyÕn víi ®å thÞ hµm sè y = x 3 − 1 t¹i ®iÓm (-1;-2).

10 − x neu x ≤ 1
5. y= x − x 2 vµ y = 
3  x − 2 neu x > 1
ln x
6. y = x −1 + ; y = x − 1; x = e
x

7. y = 1 − 1 − x2 ; y = x2
8. y = x3 ; y = 2 − x 2 ; x = 0

TÝnh thÓ tÝch khèi trßn xoay khi quay h×nh ph¼ng x¸c ®Þnh bëi:
9. y = 2 − x 2 ; y = 1 quay quanh trôc Ox.
10. y = 2 x − x 2 ; y = x quay quanh trôc Ox.
11. y = x 2 + 1; x = 0 vµ tiÕp tuyÕn víi ®å thÞ hµm sè y = x 2 + 1 t¹i ®iÓm (1;2), quay quanh trôc
Ox.
12. y = x 2 ; x = y 2 quay quanh trôc Ox.
x −1 1
13. y= ; y = ; x = 1 quay quanh trôc Ox.
x x
1
14. y = ; y = 0; x = 1; x = e quay quanh trôc Ox.
x
15. y = x 2 − 2 x ; y = 0; x = 3 quay quanh trôc Ox.

16. y = x − 1 ; y = 1 quay quanh trôc Ox.

44
Chñ ®Ò Sè phøc

Néi dung träng t©m


* M«®un cña sè phøc.
* C¸c phÐp to¸n trªn sè phøc.
* C¨n bËc hai cña sè thùc ©m.
* PT bËc hai hÖ sè thùc cã ∆ < 0.
I/ Tãm t¾t lý thuyÕt:
1. KiÕn thøc c¬ b¶n:
* Sè i: sè i lµ nghiÖm cña ph−¬ng tr×nh: x 2 + 1 = 0 . VËy: i 2 = −1
* Kh¸i niÖm sè phøc:
Sè phøc z lµ biÓu thøc cã d¹ng: z = a + bi trong ®ã: a, b ∈ ; i 2 = −1
a lµ phÇn thùc; b lµ phÇn ¶o
* Hai sè phøc b»ng nhau: y
a + bi = c + di ⇔ a = c; b = d

* BiÓu diÔn sè phøc trªn mÆt ph¼ng täa ®é:


b M
§iÓm M(a ; b) trong hÖ täa ®é Oxy ®−îc gäi lµ ®iÓm biÓu
diÔn sè phøc z = a + bi
O a x
* M«®un cña sè phøc:

Cho sè phøc z = a + bi, khi ®ã ®é dµi vect¬ OM ®−îc gäi lµ
m«®un cña sè phøc z ký hiÖu lµ z

z = ai + b = OM = a 2 + b 2

* Sè phøc liªn hîp:


Sè phøc liªn hîp cña sè phøc z = a + bi lµ z = a − bi
Chó ý: z = z vµ z = z

* C¸c phÐp to¸n trªn sè phøc:


( a + bi ) + (c + di ) = ( a + c) + (b + d )i
PhÐp céng, trõ:
( a + bi ) − (c + di ) = ( a − c) + (b − d )i

PhÐp nh©n: ( a + bi )(c + di ) = ( ac − bd ) + ( ad + bc)i

z + z = 2a
Chó ý: cho z = a + bi th×: 2
z.z = a 2 + b 2 = z

45
c + di (c + di )(a − bi ) ac + bd ad − bc
PhÐp chia: = = 2 + i (a 2 + b 2 ≠ 0)
a + bi a 2 + b2 a + b2 a 2 + b2

* C¨n bËc hai cña sè thùc ©m:


C¸c c¨n bËc hai cña sè thùc a ©m lµ: ±i a

VÝ dô: sè – 1 cã hai c¨n bËc hai lµ ±i


sè – 3 cã hai c¨n bËc hai lµ ±i 3 ...
* NghiÖm cña ph−¬ng tr×nh bËc hai víi hÖ sã thùc.
XÐt ph−¬ng tr×nh ax 2 + bx + c = 0 víi a, b, c ∈ ; a ≠ 0
∆ = b 2 − 4ac
−b ± ∆
- NÕu ∆ > 0 th× ph−¬ng tr×nh cã 2 nghiÖm thùc: x1,2 =
2a
b
- NÕu ∆ = 0 th× ph−¬ng tr×nh cã nghiÖm kÐp thùc: x=−
a

−b ± i ∆
- NÕu ∆ < 0 th× ph−¬ng tr×nh cã 2 nghiÖm phøc: x1,2 =
2a
II/ Mét sè vÝ dô minh häa:
D¹ng 1: T×m m«®un cña sè phøc
VÝ dô 1: Cho z = −2 + i 3 , t×m z .

Gi¶i:
Ta cã z = (−2) 2 + ( 3)2 = 7
VÝ dô 2: Cho z = 3i , t×m z .

Gi¶i:
Ta cã z = 02 + 32 = 3

VÝ dô 3: Cho z = − 5 , t×m z .

Gi¶i:
Ta cã z = (− 5)2 + 02 = 5

D¹ng 2: C¸c phÐp to¸n trªn sè phøc


VÝ dô 4: TÝnh (3 - 5i) + (2 + 4i)
Gi¶i:
(3 - 5i) + (2 + 4i) = (3 + 2) + (-5 + 4)i = 5 - i
VÝ dô 5: T×m x biÕt: 3x + (2 + 3i)(1 – 2i) = 5 + 4i
Gi¶i:
46
Ta cã: 3x + (2 + 3i)(1 – 2i) = 5 + 4i
⇔ 3x + (2 + 6) + (3 – 4)i = 5 + 4i
⇔ 3x + 8 – i = 5 + 4i
5
⇔ 3x = - 3 + 5i ⇔ x = −1 + i
3
3 + 2i
VÝ dô 6: Thùc hiÖn phÐp chia sau:
2 + 3i
Gi¶i:
3 + 2i (3 + 2i )(2 − 3i ) 12 − 5i 12 5
= = = − i
2 + 3i 2 2 + 32 13 12 13

VÝ dô 7: Gi¶i ph−¬ng tr×nh: ( 2 − i 3) x + i 2 = 3 + 2i 2


Gi¶i:
Ta cã: ( 2 − i 3) x + i 2 = 3 + 2i 2

⇔ ( 2 − i 3) x = 3 + i 2

3 +i 2
⇔ x=
2 −i 3

( 3 + i 2)( 2 + i 3)
⇔ x=
( 2) 2 + ( 3) 2

⇔ x=i
D¹ng 3: NghiÖm cña ph−¬ng tr×nh bËc hai víi hÖ sè thùc cã ∆ < 0
VÝ dô 8: Gi¶i ph−¬ng tr×nh x2 + x + 7 = 0
Gi¶i:
Ta cã ∆ = 1 − 4.7 = −27 < 0
VËy ph−¬ng tr×nh cã 2 nghiÖm phøc lµ:
−1 + i 27 1 3 3 −1 − i 27 1 3 3
x1 = =− + i ; x2 = =− − i
2 2 2 2 2 2
VÝ dô 9: Gi¶i ph−¬ng tr×nh x2 + 2 x + 4 = 0
Gi¶i:
Ta cã ∆ ' = 1 − 4 = −3 < 0
VËy ph−¬ng tr×nh cã 2 nghiÖm phøc lµ: x1 = −1 + i 3 ; x2 = −1 − i 3

47
PhÇn riªng Theo ch−¬ng n©ng cao

CHỦ ĐỀ PHÖÔNG PHAÙP TOÏA ÑOÄ TRONG KHOÂNG GIAN


I.TOÏA ÑOÄ CUÛA VECTÔ VAØ CUÛA ÑIEÅM
A. Lí thuyeát caàn nhôù:
1.Toïa ñoä cuûa vectô
→ → → →
Ñònh nghóa: Trong kg(Oxyz ) cho vectô u tuøy yù ,do i , j , k khoâng ñoàng phaúng
→ → → →
neân toàn taïi boä ba soá thöïc (x ; y ; z) sao u = x i + y j + z k
→ →
Boä ba soá (x ; y ; z) goïi laø toïa ñoä cuûa vectô u , kí hieäu: u = ( x ; y ; z )
→ → → → →
Vaäy u=(x;y;z)⇔ u =x i +y j +zk
→ →
Caùc tính chaát: u = ( x ; y ; z ) , v = ( x’ ; y’ ; z’ )
→ →
• u + v = ( x + x’ ; y + y’; z + z’ )
→ →
• u - v = ( x – x’ ; y – y’; z – z’ )

• k u = ( kx ; ky ; kz )
x = x'
→ → 
• u = v ⇔ y = y'
 '
z = z

2. Toïa ñoä cuûa ñieåm :


Ñònh nghóa : Trong kg(Oxyz ) cho ñieåm M tuøy yù. Toïa ñoä cuûa vectô OM ñöôïc goïi
laø toïa cuûa ñieåm M .
−→
Vaäy neáu OM = (x ; y ; z) thì boä ba soá (x ; y ; z) laø toïa ñoä cuûa ñieåm M ,

Ta vieát : M ( x ; y ; z )
→ → →
−→
M(x;y;z) ⇔ OM = x i + y j + z k

Caùc tính chaát : A ( xA ; yA ; zA ) , B ( xB ; yB ; zB ) ta coù ;


* AB = ( xB – xA ; yB – yA ; zB – zA )

* AB = ( xB − xA )2 + ( yB − yA )2 + ( zB − z A )2

48
 x A − kx B
xM = 1 − k

* MA = k MB , (k ≠ 1) ⇔  y M = y A − ky B
−→ −→

 1− k
 z A − kz B
z M = 1 − k

 x A + xB
xM = 2

* M laø trung ñieåm cuûa ñoaïn AB ⇔  y M = y A + y B
 2
 z A + zB
z M =
 2

 1
 xG = 4 ( x A + x B + xC + x D )

 1
* G(xG;yG; zG) laø troïng taâm töù dieän ABCD ⇔  y G = 4 ( y A + y B + yC + y D )

 1
zG = 4 ( z A + z B + zC + z D )

3 .Bieåu thöùc toïa ñoä cuûa tích voâ höôùng cuûa hai vectô :
→ →
Cho hai vectô a = ( x1; y1 ; z1 ) , b = ( x2 ; y2 ; z2 ) ta coù :
→ →
• a . b = x1x2 + y1y2 + z1z2
→ →
• a ⊥ b ⇔ x1x2 + y1y2 + z1z2 = 0


• | a |= x12 + y12 + z12
x1x2 + y1 y2 + z1z2
• cos ϕ =
x12 + y12 + z12 . x22 + y22 + z22
→ →
• a vaø b cuøng phöông vôùi nhau ⇔ x1: y1: z1= x2 : y2: z2
4. Tích coù höôùng cuûa hai vectô:
→ →
a. Ñònh nghóa : Cho hai vectô a = ( x1; y1 ; z1 ) , b = ( x2 ; y2 ; z2 ). Tích coù höôùng
→ → → →
cuûa hai vectô a vaø b laø moät vectô kí hieäu laø [ a , b ] vaø

49
→ →  y z1 z1 x1 x1 y 
[ a , b ] =  
1 1
; ; 
 y 2 z 2 z 2 x 2 x 2 y 2 

b. Caùc tính chaát :


→ → → → →
• a cuøng phöông vôùi b ⇔ [ a , b ] = 0
→ → → → → →
• [ a , b ]⊥ a , [ a , b ] ⊥ b
→ → → →
• |[ a , b ]| = | a |.| b |sin ϕ

c.Dieän tích tam giaùc :


Dieän tích tam giaùc ABC ñöôïc tính bôûi coâng thöùc:
1
S ∆ABC = |[AB, AC ]|
2

d.Theå tích :
• Theå tích V cuûa hình hoäp ABCD. A’B’C’D’ ñöôïc tính bôûi coâng thöùc:

V = |[AB, AD ].AA’|
• Theå tích V cuûa töù dieän ABCD ñöôïc tính bôûi coâng thöùc :
1
V= |[AB , AC ]AD |
6

e. Ñieàu kieän ñoàng phaúng cuûa ba vectô :


→ → → → → →
• Ba vectô a , b , c ñoàng phaúng ⇔ [ a , b ]. c = 0
→ → → → → →
• Ba vectô a , b , c khoâng ñoàng phaúng ⇔ [ a , b ]. c ≠ 0
  
• Boán ñieåm A,B,C,D ñoàng phaúng ⇔ AB, AC , AD ñoàng phaúng
  
• Boán ñieåm A,B,C,D khoâng ñoàng phaúng ⇔ AB, AC , AD khoâng ñoàng phaúng

50
b) Ta coù:
[a,b ] =  12 - 11 ; 1− 1 0 0 2
;  = (-3; 2; -4)
2 2 1 

→ → →
  
[ ]
a , b c = -3.2 + 2.2 + -4.(-1) ≠ 0 ⇒ 3 vectô a , b , c khoâng ñoàng phaúng .

Baøi Taäp
Baøi 1: Vieát toïa ñoä cuûa caùc vectô sau :
→ → → → → → → → → → → →
a = −2 i + 3 j − 4 k , b = 2 j− i , c = 3k , d = −k− 2 i
→ → →
Baøi2 : Cho ba vectô a = ( 2;-5 ; 3 ), b = ( 0; -2; 1) , c = (-1 ; 6; 2 ).
→ → → → 1→ → →
a) Tìm toïa ñoä cuûa vectô : u = 2 a - b . v = a + 3 b + c
2
→ → →
b) Chöùng minh raèng 3 vectô a , b , c khoâng ñoàng phaúng .
Baøi 3 : Cho ñieåm M( - 1; 2 ; 3) . Tìm toïa ñoä hình chieáu vuoâng goùc cuûa ñieåm M .
Treân truïc Ox .
Treân maët phaúng Oyz.
Baøi 4 : Cho hai ñieåm A(1 ; 2 ; 1) ,B(-2 ; 1 ; 2)
a) Tìm toïa ñoä ñieåm A’ ñoái xöùng vôùi A qua Oy.
b) Tìm toïa ñoä ñieåm B’ ñoái xöùng vôùi B qua xOy.
c) Tìm ñieåm M chia ñoaïn A’B’ theo tæ soá - 3
→ → →
Baøi 5: Cho ba vectô a = ( 0;-2 ; 4 ), b = ( 1; 3; -1) , c = (2 ; 0; 5 ).Tìm toïa ñoä cuûa :
→ → 1→ →
a) Vectô d = 4 a − b + 3 c .
3
→ → → →
b) Vectô x bieát x + 2 a = − a .
→ → → →
c) Vectô u bieát 2 a + u = 5 b

d) Tìm  a . b . c , e)  b . c . a g )  a , b . c


→ → → → → → → → →
,
     
→ → →
Baøi 6. Cho 3 vectô a = (1; m; 2), b = (m+1; 2;1 ) , c = (0 ; m-2 ; 2 ) .Xaùc đònh m ñeå 3
Vectô ñoù ñoàng phaúng .
Baøi 7. Cho 3 ñieåm A ( 3;-4;7 ),B( -5; 3; -2 ) ,C(1; 2; -3 ).
c. Xaùc ñònh ñieåm D sao cho töù giaùc ABCD laø hình bình haønh .
d. Tìm toïa ñoä giao ñieåm cuûa hai ñöôøng cheùo.
51
c. Tính dieän tích tam giaùc ABC, ñoä daøi BC töø ñoù ñöôøng cao tam giaùc ABC veõ töø A.
d. Tìm toïa ñoä troïng taâm cuûa tam giaùc ABC .
Baøi 8. Cho 4 ñieåm A( 2; 0; 0) , B( 0; 4; 0 ) , C( 0; 0; 6 ), D ( 2; 4 ;6 ).
a. Chöùng minh 4 ñieåm A, B , C , D khoâng ñoàng phaúng.Tính theå tích töù dieän
ABCD
b. Tìm toïa ñoä troïng taâm cuûa töù dieän ABCD .
c. Tính dieän tích tam giaùc ABC , töø ñoù suy ra chieàu cao cuûa töù dieän veõ töø D.
d. Tìm toïa ñoä chaân ñöôøng cao cuûa töù dieän veõ töø D .
Baøi 9. Trong khoâng gian vôùi heä toïa ñoä Oxyz cho ba ñieåm A(3;4;-1) , B(2;0;3),C(-
3;5;4)
a. Tìm ñoä daøi caùc caïnh cuûa tm giaùc ABC.
b. Tính cosin caùc goùc A,B,C .
c. Tính dieän tích tam giaùc ABC
II . PHÖÔNG TRÌNH TOÅNG QUAÙT CUÛA MAËT PHAÚNG
A. Lí thuyeát caàn nhôù :
1. Ñònh nghóa : → →
• Vectô n ≠ 0 ñöôïc goïi laø vectô phaùp tuyeán cuûa maët phaúng (α ) neáu noù naèm treân
ñöôøng thaúng vuoâng goùc vôùi ( α ).

Kí hieäu : n ⊥ ( α )
→ → →
• Trong khoâng gian vôùi heä toïa ñoä Oxyz neáu hai vectô a , b ≠ 0 ,khoâng cuøng
phöông vaø caùc ñöôøng thaúng chöùa chuùng song song hoaëc naèm trong (α ) ñöôïc
goïi laø caëp vectô chæ phöông cuûa maët phaúng ( α ).

→ → →
Chuù yù : Neáu ( α ) coù caëp vectô chæ phöông a , b thì (α ) coù moät vectô phaùp tuyeán n =
→ →
[a,b]

2.Phöông trình maët phaúng: M aët phaúng ( α ) qua M0( x0 ;y0 ; z0 ) coù vtpt n = ( A;
B; C ) coù phöông trình laø :

A ( x – x0 ) + B (y – y0) + C ( z – z0 ) = 0

52
Ví duï: Vieát phöông trình toång quaùt cuûa maët phaúng ( α ) trong caùc tröôønghôïp sau:
(α) ñi qua M (3; 2; -5 ) vaø vuoâng goùc vôùi truïc Oz .
(α) laø maët phaúng trung tröïc cuûa ñoaûn AB vôùi A( 3; -5; 4 ), B( 1 ; 3; -2 ).
Giaûi:

+ Maët phaúng(α) ñi qua M (3; 2; -5 ) vaø vuoâng goùc vôùi truïc Oz nhaän k (0;0;1) laøm
vectô phaùp tuyeán coù phöông trình toång quaùt laø:
0(x - 3) + 0(y -2) + 1(z + 5) = 0
⇔z+5=0
+ Goïi I laø trung ñieåm cuûa AB ta coù I(2;-1;1)
Ta coù AB(−2;8;−6) . Maët phaúng trung tröïc cuûa ñoaûn AB ñi qua I vaø nhaän
AB(−2;8;−6) laøm vectô phaùp tuyeán coù phöông trình toång quaùt laø:

-2(x - 2) + 8(y + 1) -6(z - 1) = 0


⇔ -2x + 8y -6z + 18 = 0

B. Phöông phaùp chung laäp phöông trình cuûa maët phaúng :


Ñeå laäp phöông trình cuûa moät maët phaúng ta caàn tìm moät ñieåm thuoäc maët phaúng
vaø vtpt cuûa noù hay tìm caëp vtcp cuûa noù
Baøi taäp
1. Vieát phöông trình maët phaúng (α) trong caùc tröôøng hôïp sau:
a. (α) ñi qua hai ñieåm M( 1; -1; 2 ) , N( 3; 1; 4 ) vaø song song vôùi truïc Oz .
b. (α) ñi qua ba ñieåm A(1; 6; 2 ), B( 5; 0; 4), C( 4; 0; 6 ) .
(α) ñi qua hai ñieåm D( 1; 0; 0 ) ,E( 0; 1; -1 ) vaø vuoâng goùc vôùi maët phaúng :
(P): x + y – z = 0 .
(α) qua ñieåm I( 3; -1; -5 ) vaø voâng goùc vôùi hai maët phaúng :
( α1): 3x –2y + 2z +5 = 0 , (α2 ): 5x – 4y + 3z +1 = 0 .
2. Trong khoâng gian vôùi heä toïa ñoä Oxyz cho ba maët phaúng :
(α1): 2x + 3y – 4 = 0 , (α2) : 2y – 3z – 5 = 0 , (α3) : 2x + y – 3z –2 = 0.
a. Vieát phöông trình maët phaúng ( α ) quañieåm M( 1;3; -4 ) giao tuyeán cuûa(α1) ,(α2)
b. Vieát phöông trình maët phaúng ( β ) qua giao tuyeán cuûa (α1) ,(α2) ñoàng thôøi vuoâng
goùc vôùi (α3) .
3. Trong khoâng gian vôùi heä toïa ñoä Oxyz cho hai ñöôøng thaúng :

53
x = 1 − t
2x − y + 4z − 5 = 0 
d1::  , (d2) :  y = 2 + 3t .
x = y − 2z −1 = 0  z = 2t

Vieát phöông trình maët phaúng (α) qua (d1) vaø song song vôùi (d2).
Vieát phöông trình maët phaúng (α1) qua M (1 ;–3; 5 ) vaø song song vôùi hai
ñöôøng thaúng (d1), (d2) .
4. Trong khoâng gian vôùi heä toïa ñoä Oxyz.Cho ñieåm M( 2;-1 ; 1) vaø ñöôøng thaúng
2 x − y + 3 z − 8 = 0
d:  . Vieát phöông trình maët phaúng ñi qua M vaø vuoâng
x + 2 y − 2z + 3 = 0
goùc vôùi ñöôøng thaúng d.
x y +1 z − 2
5. Vieát phöông trình maët phaúng (P) chöùa ñöôøng thaúng d: = = vaø
1 2 −2
vuoâng goùcvôùi maët phaúng (Q): 2x – 3y + z + 3 = 0
III. ÑÖÔØNG THAÚNG
A. Lí thuyeát caàn nhôù
→ →
Vectô u ≠ 0 naèm treân ñöôøng thaúng song song hoaëc truøng vôùi ñöôõng thaúng (d)
goïi laø vectô chæ phöông cuûa ñöôøng thaúng (d).

Ñöôøng thaúng (d) ñi qua ñieåm M0( x0; y0 ; z0 ) coù vectô chæ phöông u = ( a; b; c)
 x = x 0 + at

coù phöông trình tham soá laø :  y = y 0 + bt t∈R
 z = z + ct
 0

x − x0 y − y0 z − z0
Phöông trình chính taéc : = = .
a b c
 Ax + By + Cz + D = 0
• Phöông trình toång quaùt cuûa ñöôøng thaúng :  (1) trong ñoù
 A' x + B ' y + C ' z + D ' = 0
A2+B2+C2 ≠ 0, A’2+B’2+C’2≠ 0 , A:B:C ≠ A’:B’:C’.
Chuù yù: Neáu ñöôøng thaúng coù phöông trình daïng (1) thì noù coù moät vectô chæ phöông
→ B C C A A B
u=( ; ; )
B ' C ' C ' A' A'B '

B. Phöông phaùp chung ñeå laäp phöông trình cuûa ñöôøng thaúng:
Ñeå laäp phöông trình cuûa moät ñöôøng thaúng ta söû duïng moät trong hai caùch sau:
• Tìm vectô chæ phöông cuûa ñöôøng thaúng vaø moät ñieåm thuoäc ñöôøng thaúng.

54
• Vieát phöông trình hai maët phaúng phaân bieät vaø chöùa ñöôøng thaúng ñoù.
Chuù yù : Hai ñöôøng thaúng song song coù cuøng vectô chæ phöông.
Neáu ñöôøng thaúng vuoâng goùc vôùi maët phaúng thì noù nhaän vtpt cuûa maët phaúng
laøm vtcp.
C. Moät soá caùch vieát phöông trình ñöôøng thaúng thöôøng gaëp:
1/ Baøi toaùn 1:Vieát phöông trình hình chieáu voâng goùc cuûa ñöôøng thaúng (d) treân maët
phaúng (α ).
Caùch giaûi :
• Vieát phöông trình maët phaúng (β ) qua ñöôøng thaúng (d ) vaø vuoâng goùc vôùi (α ).
( Maët phaúng (β ) nhaän vtcp cuûa(d) vaø vtpt cuûa (α ) laøm caëp vtcp )
• Hình chieáu vuoâng goùc (d’) cuûa (d) treân (α ) laø giao tuyeán cuûa (α ) vaø (β ).
2/ Baøi toaùn 2: Vieát phöông trính ñöôøng thaúng (d) ñi qua ñieåm M vaø caét caû hai
ñöôøng thaúng (d1) , (d2) cho tröôùc .( M ∉ (d1),(d2)) .
Caùch giaûi :
• Vieát phöông trình maët phaúng ( M,(d1))
• Vieát phöông trình maët phaúng (M,(d2))
• (d) = (M,(d1)) ∩ (M,(d2)).
3/ Baøi toaùn 3: Vieát phöông trình ñöôøng thaúng (d ) qua M caét ñöôøng thaúng (d1) vaø
vuoâng goùc vôùi (d2).
Caùch giaûi :
• Vieát phöông trình maët phaúng (α ) qua M vaø (d1).
• Vieát phöông trình maët phaúng (β ) qua M vaø (β )⊥ (d2).
• (d) = (α) ∩ (β).
4/ Baøi toaùn 4: Vieát phöông trình ñöôøng thaúng (d ) ñi qua ñieåm M caét ñöôøng thaúng
( ∆ ) vaø vuoâng goùc vôùi ( ∆ ).
Caùch giaûi:
• Vieát phöông trình maët phaúng (α) qua M vaø vuoâng goùc vôùi ( ∆ ).
• Vieát phöông trình maët phaúng (β) qua M vaø ( ∆ ).
• (d) = (α) ∩ (β) .
Ghi chuù :Ta coù theå giaûi baøi toaùn nhö sau.
• Vieát phöông trình maët phaúng (α) qua M vaø vuoâng goùc vôùi ( ∆ ).
• Tìm giao ñieåm N cuûa ( ∆ ) vaø(α ).

55
• Vieát phöông trình ñöôøng thaúng MN ñoù laø ñöôøng thaúng (d) caàn tìm.
5/ Baøi toaùn 5: Cho ñöôøng thaúng ( ∆ ) vaø maët phaúng (α ) caét nhau taïi ñieåm M .Vieát
phöông tình ñöôøng thaúng (d) ñi qua M naèm trong (α ) vaø (d)⊥ ( ∆ ).
Caùch giaûi :
• Vieát phöông trình maët phaúng (β) qua M vaø (β)Vuoâng goùc vôùi (d) .
• (d) = (α)∩ (β).

6/ Baøi toaùn 6 : Vieát phöông trình ñöôøng thaúng ( ∆ ) coù vtcp u vaø caét hai ñöôøng thaúng
(d1) vaø (d2) cho tröôùc.
Caùch giaûi :

Vieát phöông trình maët phaúng (α) qua (d1) vaø nhaän u laøm moät vtcp.

Vieát phöông trình maët phaúng (α) qua (d2) vaø nhaän u laøm moät vtcp.
(c) = (α)∩ (β).
Ví duï: Vieát phöông trình tham soá vaø chính taéc cuûa ñöôøng thaúng ( ∆ ):
d. Qua hai ñieåm M( 2; -3; 5), N( 1; -2; 3).
e. Qua A(1; -1; 3) vaø song song vôùi BC trong ñoù B(1; 2; 0 ),C(-1; 1; 2)
f. Qua D(3; 1; -2) vaø vuoâng goùc vôùi maët phaúng 3x + 4y – 2z +5 = 0
Giaûi:
a. Ñöôøng thaúng ( ∆ ) ñi qua ñieåm M( 2; -3; 5) nhaän vectô MN (−1;1;−2) laøm
vectô chæ phöông coù phöông trình :
x = 2 − t

+ Phöông trình tham soá:  y = −3 + t , t ∈ R
 z = 5 − 2t

x −2 y +3 z −5
+ Phöông trình chính taéc: = =
−1 1 −2
b. . Ñöôøng thaúng ( ∆ ) ñi qua ñieåm A( 1; -1; 3) song song vôùi BC nhaän vectô
BC (−2;−1;2) laøm vectô chæ phöông coù phöông trình :

 x = 1 − 2t

+ Phöông trình tham soá:  y = −1 − t , t ∈ R
 z = 3 + 2t

x −1 y +1 z − 3
+ Phöông trình chính taéc: = =
−2 −1 2
c. Ñöôøng thaúng ( ∆ ) ñi qua ñieåm D( 3; 1; -2) vaø vuoâng goùc vôùi maët phaúng

56

3x + 4y – 2z +5 = 0 nhaän vectô phaùp tuyeán cuûa maët phaúng n (3;4;−2) laøm
vectô chæ phöông coù phöông trình:
 x = 3 + 3t

+ Phöông trình tham soá:  y = 1 + 4t , t ∈ R
 z = −2 − 2t

x − 3 y −1 z + 2
+ Phöông trình chính taéc: = =
3 4 −2
D. Baøi taäp :
3x − 2 y + z − 10 = 0
1. Cho ñöôøng thaúng (d) coù phöông trình toång quaùt  .
x + 2 y − 4z + 2 = 0
Haõy vieát phöông trình tham soá vaø phöông trình chính taéc cuûa (d).
x − 2z = 0
2. Cho ñöôøng thaúng (d) :  vaø maët phaúng (α): x –2y + z +5 = 0.
3x − 2y + z −3 = 0
Vieát phöông trình hình chieáu vuoâng goùc cuûa (d) treân (α).
x −1 x + y − z + 2 = 0
3. Cho hai ñöôøng thaúng: (d1) = y+2= z , (d2):  .
3 x + 1 = 0
a.Vieát phöông trình ñöôøng thaúng (d) qua A( 0; 1; 1) vuoâng goùc vôùi (d1) vaø caét (d2).
b. Vieát phöông trình ñöôøng thaúng (∆ )Qua ñieåm M(1; 0; -2 )vaø vuoâng goùc vôùi
hai ñöôøng thaúng (d1), (d2).

4. Vieát phöông trình ñöôøng thaúng qua A( 3; -2; - 4),song song vôùi maëtt phaúng :
x − 2 y + 4 z −1
3x – 2y – 3z – 7 = 0 ñoàng thôøi caét ñöôøng thaúng (d): = =
3 −2 2
5. Laäp phöông trình ñöôøng thaúng vuoâng goùc vôùi maët phaúng Oxy vaø caét caû hai ñöôøng
x = t  x = 1 − 2t
 
thaúng : (d1):  y = −4 + t , (d2):  y = −3 + t .
z = 3 − t  z = 4 − 5t
 
IV. VÒ TRÍ TÖÔNG ÑOÁI CUÛA CAÙC ÑÖÔØNG THAÚNG VAØ MAËT PHAÚNG.
A. LÍ THUYEÁT :
1/ Vò trí töông ñoái cuûa hai ñöôøng thaúng:
x − x0 y − y 0 z − z 0 x − x0' y − y 0' z − z 0'
Cho hai ñöôøng thaúng : (d) : = = ,( d’ ): = =
a b c a' b' c'

(d) qua M0(x0 ;y0 ;z0) ,coù VTCP u = ( a; b; c)

57

(d’) qua M’0(x’0 ;y’0 ;z’0) ,coù VTCP u ' = ( a’; b’; c’)
→ → − −−→
’ ' '
a. (d) vaø (d ) ñoàng phaúng ⇔ [u , u ]. M 0 M 0 = 0
→ → −−→
' '
b. (d) vaø (d’) caét nhau ⇔ [u , u ]. M 0 M 0 = 0 vaø a:b:c ≠ a’:b’:c’

c. (d)//(d’) ⇔ a:b:c = a’:b’:c’≠ (x’0 – x0 ):(y’0 – y0) :(z’0 – z0)


d. (d) ≡ (d’) ⇔ a:b:c = a’:b’:c’ = (x’0 – x0 ):(y’0 – y0) :(z’0 – z0)
→ → −− −→
' '
e. (d) vaø (d’) cheùo nhau ⇔ [u , u ]. M 0 M 0 ≠ 0

Ví duï: Xeùt vò trí töông ñoái cuûa caëp ñöôøng thaúng sau

x −1 y + 2 z x + y − z + 2 = 0
d: = = vaø d’ 
3 1 1 x + 1 = 0

Giaûi: a. Ñöôøng thaúng (d) ñi qua ñieåm M(1; -2; 0) coù vectô chæ phöông u (3;1;1) . Ñöôøng

thaúng (d’) ñi qua ñieåm M’(-1; 0; 1) coù vectô chæ phöông u , (0;−1;−1) .
 
Tacoù: + [u , u ′] =(0; 3; -3), MM ′ (-4; -1; 0)
 
+ [u , u ′] . MM ′ =0(-4) +3(-1) +(-3)0 = -3 ≠ 0 ⇒ d vaø d’ cheùo nhau

Baøi taäp
Baøi 1: Xeùt vò trí töông ñoái cuûa caùc caëp ñöôøng thaúng sau
 x + 2 y − 3z + 1 = 0 x−2 y z+3
a/ d:  vaø d’: = =
x + y + 2z −1 = 0 7 −5 −1
x −1 y − 2 z − 3 x−7 y −6 z −5
b/ d: = = vaø d’: = =
9 6 3 6 4 2
Baøi 2: Xeùt vò trí töông ñoái cuûa caùc ñöôøng thaúng sau :
x − y + z − 1 = 0
d’: 
2x − y + 1 = 0
a) d:  vaø
2 x + y + 1 = 0 3x + y − z + 3 = 0
3 x + 2 y − 25 = 0 x−2 y z +1
b) d:  vaø d’: = =
2 x + z − 14 = 0 4 −6 −8

x −1 y − 2 z − 3 x−7 y −6 z −5
c) d: = = vaø d’: = =
9 6 3 6 4 2
 x = −2t
x −1 y − 2 z 
d) = = vaø d’ :  y = − 5 + 3 t .
2 −2 1 z = 4

58
2/ Vò trí töông ñoái cuûa hai maët phaúng :
Cho hai mp (α ) , ( β ) lần lượt có phương trình:

(α ) : Ax+By+Cz+D=0
( β ):A’x+B’y+C’z+D=0
a) ( α ) cắt ( β )

⇔ A : B : C ≠ A': B ': C '


A B C D
b) (α ) //( β ) ⇔ = = ≠
A′ B ′ C ′ D ′
A B C D
c) (α ) ≡ ( β ) ⇔ = = =
A' B ' C ' D '
Chú ý: Điều kiện vuông góc giữa 2 mp:
(α ) ⊥ ( β ) ⇔ AA '+ BB '+ CC ' = 0
BAØI TAÄP
Baøi 1: Xeùt vò trí töông ñoái cuûa caùc caëp maët phaúng sau
a. ( α ): x + 2y – z + 5 = 0 vaø ( β ): 2x + 3y -7z – 4 = 0
b. ( α ) : x – 2y + z + 3 = 0 vaø ( β ): 2x -y + 4z – 2 = 0
c. ( α ): x + y + z - 1 = 0 vaø ( β ): 2x + 2y -2z + 3 = 0
d. ( α ): 3x - 2y –3 z + 5 = 0 vaø ( β ): 9x - 6y -9z – 5 = 0
e.( α ): x - y + 2z -4 = 0 vaø ( β ): 10x - 6y + 20z – 40 = 0

Baøi 2: Xeùt vò trí töông ñoái cuûa caùc caëp maët phaúng sau :
a) 2x – 3y + 4z – 5 = 0 vaø 3x – y +z – 1 = 0 .
b) – x +y – z + 4 = 0 vaø 2x – 2y + 2z – 7 = 0.
c) x + y + z – 3 = 0 vaø 2x – 2 y + 2 z – 3 = 0.
d) 3x + 3y – 3z – 12 = 0 vaø 4 x + 4y – 4z – 16 = 0.
Baøi 3 : Cho hai maët phaúng coù phöông trình :(m2–5)x – 2y + mz + m – 5 = 0 vaø x + 2y
– 3nz +3 = 0 .
Tìm m , n ñeå hai maët phaúng :
a) Song song vôùi nhau .
b) Truøng nhau .
c) Caét nhau .
Baøi 4 : Cho hai maët phaúng : 3x – (m – 3)y +2z – 5 = 0 vaø (m + 2)x – 2y + mz – 10 = 0
.Tìm m ñeå
59
d) Hai maët phaúng song song vôùi nhau .
e) Hai maët phaúng truøng nhau .
f) Hai maët phaúng caét nhau .
V. KHOAÛNG CAÙCH, GOÙC
A. LÍ THUYẾT :
1. Khoaûng caùch töø moät ñieåm tôùi moät maët phaúng
Khoaûng caùch töø ñieåm M0(x0; y0; z0) ñeán maët phaúng (α): Ax + By + Cz + D = 0
Ax0 + By 0 + Cz 0 + D
Kí hieäu: d(M0;(α)) =
A2 + B 2 + C 2

2. Khoaûng caùch töø moät ñieåm tôùi moätñöôøng thaúng


Khoaûng caùch töø ñieåm M ñeán ñöôøng thaúng (∆ ) ñi qua ñieåm M0 vaø coù vectô chæ

phöông u ñöôïc xaùc ñònh theo coâng thöùc:
[M M , u]
0
d(M, ∆) = 
u

3. Khoaûng caùch giöõa hai ñöôøng thaúng cheùo nhau



Cho hai ñöôøng thaúng d1 ñi qua ñieåm M1 coù vecytô chæ phöông u1 vaø ñöôøng thaúng

d2 ñi qua ñieåm M2 coù vecytô chæ phöông u 2 . Goïi h laø khoaûng caùch giöõa d1 vaø d2

[u1 , u2 ].M 1 M 2


ta coù: h=
[u1 , u2 ]
B.BAØI TAÂP:
1/ Tính khoaûng caùch töø caùc ñieåm M1(1;-3;4) , M2( 0;4 ;1) , M3( 2;-1;0 ) ñeán maët
phaúng (α) : 2x –2y + z – 5 = 0
2/ Tính khoaûng caùch töø ñieåm A(1;1;3) tôùi ñöôøng thaúng
x+2 y −1 z +1
∆: = =
1 2 −3
3/ Tính khoaûng caùch giöõa hai ñöôøng thaúng cheùo nhau :
x + y − z + 5 = 0 x −1 y + 2 z − 3
(∆1):  vaø (∆2): = =
2 x − y + 1 = 0 1 1 −1
x +1 y z −1
4/ Cho ñöôøng thaúng d: = = vaø maët phaúng (α):x+ y + 2z – 4 = 0 .
2 −1 −1
Tính goùc giöõa d vaø (α)
5/ Tìm treân Oz ñieåm M caùch ñeàu ñieåm A( 2; 3; -1 )vaø maët phaúng:x + 3y +z –17 = 0

60
 x = 1 + 2t

6/ Cho ñöôøng thaúng (d):  y = 2 − t vaø maët phaúng (α) : 2x – y – 2z +1 = 0.
 z = 3t

Tìm caùc ñieåm M ∈ (d) sao cho khoaûng caùch töø M ñeán (α) baèng 3 .
x−2 y −3 z + 4 x +1 y − 4 z − 4
7/ Cho hai ñöôøng thaúng (d1): = = vaø (d2): = =
2 3 −5 3 −2 −1
Tìm hai ñieåm M,N laàn löôït treân (d1) vaø (d2) sao cho ñoä daøi ñoaïn MN noû nhaát.
VI. MAËT CAÀU
A.Lí thuyeát caàn nhôù:
1/ Phöông trình Maët caàu:
a. Maët caàu (S) coù taâm I(a;b;c) coù baùn kính R coù phöông trình laø:
( x- a )2 + ( y - b )2 + ( z - c )2 = R2
b. Phöông trình : x2+y2+z2 – 2ax – 2by – 2cz + d = 0 ,a2+b2+c2- d > 0
laø phöông trình cuûa maët caàu coù taâm I(a;b;c) , baùn kính R = a 2 + b 2 + c 2 − d
2/ Vò trí töông ñoái cuûa maët caàu vôùi maët phaúng :
Cho mp(α) :Ax + By + Cz + D = 0 vaø maët caàu (S) coù phöông trình:
(x – a)2+ (y – b)2 + (z – c)2 = R2
Goïi H laø hình chieáu vuoâng goùc cuûa taâm I(a;b;c) cuûa (S) treân (α)
Aa + Bb + Cc + D
Vaäy IH = d ( I , α ) =
A2 + B 2 + C 2

a. Neáu IH < R thì (α) caét (S) theo giao tuyeán laø moät ñöôøng troøn ( C)coù taâm H
,coù baùn r = R 2 − IH 2
( x − a ) 2 + ( y − b) 2 + ( z − c) 2 = R 2
Phöông trình cuûa ñöôøng troøn (C) : 
 Ax + By + Cz + D = 0
b. Neáu IH = R thì (α) tieáp xuùc vôùi (S) taïi H .(α) goïi laø maët tieáp dieän cuûa mc(S)
c. Neáu IH > R thì (α) vaø (S) khoâng coù ñieåm chung
B. Caùc daïng baøi taäp thöôøng gaëp:
1/ Tìm taâm vaø baùn kính cuûa maët caàu sau :
a) x2 + y2 + z2 – 8x + 2y +1 = 0
b) x2 + y2 + z2 + 4x + 8y – 2z – 4 = 0
c) 3x2 + 3y2 + 3z2 + 6x – 9y + 12z – 4 = 0
2/ Vieát phöông trình maët caàu (S) trong caùc tröôøng hôïp sau :
61
a) (S) coù taâm I ( 1; -2 ; 3 ) vaø ñi qua ñieåm M( 3 ; 2 ; 4 ).
b) (S) coù ñöôøng kính AB vôùi A(1; 4 ; 5), B ( 3; -2; 7 ).
c) (S) coù taâm I( 0 ; 4; 3 ) vaø tieáp xuùc vôùi maët phaúng (α) : 2x + y – 2z + 8 = 0
d) (S) ngoaïi tieáp töù dieän ABCD vôùi A( 3; 2; 6 ), B( 3; -1; 0 ),C( 0; -7; 3 ),D(-2;1; -1).
3/ Laäp phöông trình maët caàu (S) ñi qua ba ñieåm A( 1; 2; - 4 ) , B( 1; - 3; 1 )
C( 2; 2; 3 ) vaø coù taâm I naèm treân maët phaúng Oxy.
4/ Trong khoâng gian vôùi heä toïa ñoä Oxyz cho maët caàu (S): x2+y2+z2 = 4vaø maët phaúng
(α): x + z = 2.
Chöùng minh raèng mp(α) caét maët caàu (S).
Xaùc ñònh taâm vaø tính baùn kính cuûa ñöôøng troøn (C) laø giao tuyeán cuûa (α) vôùi (S).
x = −t
5/ Cho (d) :  y = − 1 + 2 t vaø maët phaúng (α) :2x - y – 2z –2 = 0 .Vieát phöông trình maët
z = 2 + t

caàu coù taâm
I ∈ (d) caùch (α) moät ñoaïn baèng 2 vaø caét maët phaúng (α) theo giao tuyeán laø
ñöôøngtroøn coù baùn kính baèng 3 .
x y −1 z +1
6/ Cho ñöôøng thaúng (d): = = vaø hai maët phaúng(α): x+ y -2z +5 = 0 ,
2 1 2
(β) : 2x – y + z + 2 = 0 .Vieát phöông trình maët caàu coù taâm treân (d) vaø tieáp xuùc vôùi
hai maët phaúng (α) , (β).
 x 2 + y 2 + z 2 − 4 x + 6 y + 6 z + 17 = 0
7/ Cho döôøng troøn ( C ) : 
x − 2 y + 2z + 1 = 0
a) Tìm taâm vaø baùn kinh cuûa ( C ).
b) Laäp phöông trình maët caàu (S) chöùa ñöôøng troøn ( C ) vaø coù taâm treân maët
phaúng
(p): x + y + x + 3 = 0.
8/ Laäp phöông trình maët tieáp dieän cuûa maët caàu (S):x2+y2+z2 – 6x– 2y+4z+5 =0.
Tại ñieåm M(4; 3; 0 ).
9/ Laäp phöông trình maët (α) tieáp xuùc vôùi maët caàu x2+y2+z2 –26x– 2y-2z –22= 0
bieát (α) song song vôùi ( β ): 3x – 2y + 6z +14 = 0.
 x = 4 + 4t

10/ Vieát phöông trình maët phaúng chöùa ñöôøng thaúng (d):  y = 1 + 3t vaø tieáp xuùc vôùi
z = 1 + t

maët caàu (S) : x2 + y2+ z2 – 2x + 6y+ 2z + 8 = 0
62
BÀI TẬP
Baøi 1 : Trong heä toïa ñoä Oxyz cho ba ñieåm A(1;2 ; -3) , B(3 ; 2 ; 0) , C ( -4; 2 ; 5).
c) Chöùng minh A , B ,C laø ba ñænh cuûa moät tam giaùc .
d) Tìm toïa ñoä ñieåm D sao cho töù giaùc ABCD laø hình bình haønh .
e) Tìm a , b ñeå ñieåm M(a+2 ;2b – 1 ; 1) thuoäc ñöôøng thaúng AC.
Baøi 2: Cho boán ñieåm A(-3 ; 5 ;15) , B(0 ;0 ;7) , C (- 4 ; 2 ; 5) , D(4 ;-3 ; 0) .Chöùng
minh raèng hai ñöôøng thaúng AB vaø CD caét nhau .
Baøi 3 : Cho tam giaùc ABC coù A(1 ; 0 ; 3) ,B( 2 ; 2 ;4) , C( 0 ;3 ; -2).
Chöùng minh raèng tam giaùc ABC vuoâng taïi A, töø ñoù tìm taâm vaø baùn kính cuûa
ñöôøng troøn ngoaïi tieáp tam giaùc ABC .
Tính goùc C cuûa tam giaùc .
Baøi4: Cho ba ñieåm A(2 ; 1 ; 0) ,B(0 ; 0 ; 1) ,C(1 ; 1 ; 2 ) . Tính dieän tích tam giaùc
ABC, töø ñoù suy ra ñoä daøi ñöôøng cao veû töø A cuûa tam giaùc .
Baøi 5: Cho tam giaùc ABC vôùi A(1 ; 1 ; 0) , B(3 ; -1 ; 1) ,C(5 ; 1 ; 3).Tính ñoä daøi ñöôøng
phaân giaùc trong cuûa goùc A.
Baøi 6: Cho boá ñieåm A(0 ; -1 ; 0) , B(0 ; 0 ; 2) ,C( 1 ; 0 ; 0) , D(-1 ; 1 ; -2) .
a) Chöùng minh raèng A, B, C , D laø 4 ñænh cuûa töù dieän .
b) Chöùng minh raèng AC vuoâng goùc vôùi BD .
c) Tính goùc taïo bôùi hai ñöôøng thaúng AB vaø CD .
d) Tính theå tích cuûa töù dòen vaø ñoä daøi ñöôøng cao cuûa töù dieän haï töø A.
Baøi 7 : Cho ba ñieåm A(2 ; 0 ; 0) , B(0 ; 2 ;0) , C( 0 ; 0 ; 2) ,D(a ; a ; a) vôùi a laø haèng soá
a ≠ 0 . Chöùng minh raèng OD vuoâng goùc vôùi maët phaúng (ABC) vôùi moïi a.
Baøi 8: Cho hình hoäp ABCD.A’B’C’D’ coù A(0 ; 0 ; 0) ,B(1 ; 0 ; 0) ,C (0 ; 2 ;0) , A’( 0 ;
0 ; 3).
a) Tìm toïa ñoä caùc ñænh coøn laïi cuûa hình hoäp .
b) Goi M,N,P,Q laàn löôït laø trung ñieåm cuûa A’B’, BC , CD, DD’ . Chöùng minh
raèng M,N,P,Q ñoàng phaúng .
c) Tính khoaûng caùch töø C’ ñeán maët phaúng (MNPQ)
Baøi 9 : Cho hình laäp phöông ABCD.A’B’C’D’ coù caïnh baèng a . Goïi M,N laàn löôït laø
trung ñieåm cuûa A’D’ vaø B’B .
a) Chöùng minh raèng MN vuoâng goùc vôùi AC’ .
b) Chöùng minh raèng AC’ vuoâng goùc vôùi maët phaúng (A’BD).
c) Tính goùc giöõa MN vaø CC’.
63
Baøi 10 : Vieát phöông trình toång quaùt cuûa maët phaúng ( α ) trong caùc tröôønghôïp sau:
a) (α) ñi qua A (1; 0; 2 ) vaø vuoâng goùc vôùi maët phaúng Oxy .
b) (α) ñi qua M(2 ; -1 ; -3) vaø vuoâng goùc vôùi truïcc Ox .
c) (α) laø maët trung tröïc cuûa ñoaïn AB vôùi A(1; 3; 2 ), B(-1 ; 1; 0 ).
d) (α) qua I(-1; 2;4 ) vaø song song vôùi maët phaúng 2x – 3y + 5z – 1 = 0.
Baøi 11 : Trong khoâng gian vôùi heä toïa ñoä Oxyz , cho ba ñieåm A(1; -1;-3) ,B(2 ;1 ; -2) ,
C(-5 ; 2 ; -6) .
d) Chöùng minh A, B , C laø ba ñænh cuûa tam giaùc .
e) Tính ñoä daøi phaân giaùc ngoaøi goùc A cuûa tam giaùc ABC.
f) Tìm toïa ñoä tröïc taâm cuûa tam giaùc ABC.
Baøi 12:Cho maët phaúng (P) : 2x + 5y – 7x +1 = 0 .
a) Haõy xaùc ñònh vectô phaùp tuyeán cuûa (P).
b) Xaùc ñònh m ñeå ñieåm A(2m – 1 ; m +2 ; m – 1) naèm treân (P).
c) Tìm toïa ñoä giao ñieåm cuûa (P) vôùi caùc truïc toïa ñoä .
d) Tính theå tích phaàn khoâng gian giôùi haïn bôûi (P) vaø caùc maët phaúng toïa ñoä .
Baøi 13 : Vieát phöông trình maët phaúng :
a) Ñi qua A( 1 ; 0 ; 2) vaø song song vôùi maët phaúng xOy.
b) Ñi qua M(2 ;-1 ; -3) vaø vuoâng goùc vôùi truïc Ox .
c) Ñi qua I( -1 ; 2 ; 4) vaø song song vôùi maët phaúng (P): 2x – 3y + 5z – 1 = 0 .
d) (α ) laø maët trung töïc cuûa ñoaïn AB vôùi A(1 ; 2 ; 3) , B(-1 ; 1 ; 0).
e) (β ) ñi qua ba ñieåm A(-1 ; 2 ; 3) ,B(2 ; -4 ; 3) , C(4 ; 5 ; 6).
f) Ñi qua hình chieáu cuûa ñieåm N( 1 ; -3 ; 1) treân caùc truïc toïa ñoä .
Baøi 14:Cho ñieåm M(1 ; 2 ; 3) .
a) Vieát phöông trình maët phaúng (α ) vaø caét ba truïc toïa ñoä taïi A, B, C sao cho
M laø troïng taâm cuûa tam giaùc ABC .
b) Vieát phöông trình maët phaúng (β ) vaø caét ba truïc toïa ñoä taïi N, P , Q sao cho
M laø tröïc taâm cuûa tam giaùc ABC .
c) Vieát phöông trình maët phaúng ñi qua M caét ba truïc toïa ñoä taïi ba ñieåm caùch
ñeàu goác toïa ñoä.
Baøi 15 :Vieát phöông trình maët phaúng :
a) Ñi qua hai ñieåm A(1 ;1 ;0), B(-1 ; 2 ; 7) vaø vuoâng goùc vôùi maët phaúng (α)
2x–3y+z–7 = 0.

64
b) Ñi qua M(0 ;2; -1), song song vôùi truïc Ox vaø vuoâng goùc vôùi maët phaúng (β)
x – y +z = 0 .
c) Ñi qua N(-3;0;1) vaø vuoâng goùc vôùi hai maët phaúng (P):2x–3y+z –2 = 0 ;(Q):x
+ 5y–2z = 0
Baøi 16: Cho töù dieän ABCD coù A(5 ; 1 ; 3) ,B(1 ; 6 ; 2) , C(5 ; 0 ; 4) ,D(4 ; 0 ;6) .
a) Vieát phöông trình maët phaúng (BCD).
b) Vieát phöông trình maët phaúng ñi qua AB vaø song song vôùi CD .
c) Goïi G laø troïng taâm cuûa tam giaùc BCD . Vieát phöông trình maët phaúng ñi qua
G vaø song song vôùi maët phaúng (ABC ) .
Baøi 17 : Vieát phöông trình maët phaúng :
Ñi qua A(1 ; 2 ; 1 ) vaø chöùa truïc Oy .
Ñi qua giao tuyeán cuûa hai maêt phaúng : x – 3z +1 = 0 , 2y +3z – 5 = 0 vaø vuoâng
goùc vôùi maët phaúng 2x – y – 1 = 0 .
Ñi qua giao tuyeán cuûa hai maêt phaúng 3x – y + 3z +8 = 0 , -2x – y +z +2 = 0 vaø
song song vôùi maët phaúng x – y – 1 = 0.
Baøi 18 : Vieát phöo8ng trình tham soá, ptct cuûa ñöôøng thaúng ñi qua hai ñieåm A(-1 ; 4 ;
3) , B(2 ; 1 ; 1).
Baøi 19 : Vieát phöông trình toång quaùt cuûa ñöôøng thaúng laø giao tuyeán cuûa maët phaúng ñi
qua
x +1 y − 4 z − 3
M(2 ; 5 ; -3) vaø chöùa ñöôøng thaúng = = vaø maët phaúng Oxy.
3 −3 −2
Baøi 20 : Vieát phöông trình chính taéc cuûa ñöôøng thaúng :
x + y − z + 5 = 0
a) Coù phöông trình toång quaùt :  .
2 x − y + 1 = 0
 x = 1 + 3t

b) Ñi qua ñieåm M( 1 ; - 2 ; 3) vaø song song vôùi ñöôøng thaúng :  y = −3 − t
 z = 4t

c) Ñi qua ñieåm N( 2 ; 3 ; - 4) vaø vöoâng goùc vôùi maët phaúng x -2y + z – 6 = 0
3 x − 2 y + z + 1 = 0
d) Ñi qua ñieåm A( - 2 ; 5 ; 1 ) vaø song vôùi ñöôøng thaúng 
x + y − 2z − 5 = 0
e) Ñöôøng thaúng caàn tìm laø giao tuyeán cuûa (P): x -2y + 3z – 1 = 0 vôùi maët
phaúng yOz .
5 x − 3 y + 2 z − 5 = 0
Baøi 21 :Chöùng minh raèng ñöôøng thaúng d:  naèm trong maët phaúng
2 x − y − z − 1 = 0
(P):4x – 3y +7z = 0.
65
Baøi 22 :Vieát phöông trình maët phaúng (P) trong caùc tröôøng hôïp sau :
 x = 1 + 3t

a) (P) chöùa ñöôøng thaúng d vaø song song vôùi d’ bieát :d:  y = 3 + 2t vaø
 z = −2 − t

2 x − y + z − 3 = 0
d’:  .
x + 2 y − z − 5 = 0

b) (P) chöùa ñöôøng thaúng d vaø (P) vuoâng goùc vôùi maët phaúng (Q) bieát :
x −1 y + 2 z − 2
d: = = vaø (Q) : 3x +2y – z – 5 = 0 .
2 −3 2
Baøi 23 :Vieát phöông trình ñöôøng thaúng d song song vôùi hai maët phaúng (P) : 3x + 12y –
3z – 5 = 0 ;
(Q) : 3x – 4y +9z +7 = 0 vaø caét hai hai ñöôøng thaúng :d1:
x + 5 y − 3 z +1 x − 3 y +1 z − 2
= = ,d2: = =
2 −4 3 −2 3 4
Baøi 24: Vieát phöông trình ñöôøng thaúng d’ laø hình chieáu vuoâng goùc cuûa ñöôøng thaúng d
treân maët phaúng (P)
 x = 12 + 4 t

vôùi : d:  y = 3 + 3 t vaø (P) :3x + 5y – z – 2 = 0 .
z = 1 + t

Baøi 25: Vieát phöông trình ñöôøng thaúng d’ ñi qua giao ñieåm cuûa ñöôøng thaúng d vaø maët
x −1 y z + 2
phaúng (P) bieát : d : = = vaø (P) 2x +y + z – 1 = 0
2 1 −3
Baøi 26 Chöùng minh hai ñöôøng thaúng sau ñaây song vôù nhau vaø vieát phöông trình maët
phaúng chöùa hai ñöôøng thaúng ñoù
x + 2 y −1 z x + y − z = 0
d: = = vaø d’: 
3 −2 1 x − y + z − 8 = 0
Baøi 27 : Vieát phöông trình ñöôøng vuoâng goùc chung cuûa hai ñöôøng thaúng cheùo nhau
sau :
 x = 1 − 2t  x = 2t x = 1 + t
   x + y − z = 0
a) d1:  y = 3 + t , d2 :  y = 1 + t b) d1 :  y = −2 − t , d2: 
 z = −2 − t  z = 3 − 2t z = 3 − t x − y + z − 8 = 0
  
Baøi 28: Trong khoâng gian vôùi heä truïc toïa Oxyz cho ñieåm M(1 ; -2 ; 3) . Tính khoaûng
caùch töø M ñeán :
a) Maët phaúng Oyz .
b) Maët phaúng (P): x – 2y – 2z + 3 = 0.

66
x − 3 y + z = 0
c) Ñöôøng thaúng d :  .
x + y − z + 2 = 0
Baøi 29 : Trong khoâng gian vôùi heä truïc toïa ñoâ Oxyz cho hai ñöôøng thaúng :
x −1 y + 2 z + 2 x +1 y z − 3
d1: = = , d2 : = = .
3 1 4 2 3 −1
a) Chöùng minh hai ñöôøng thaúng d1 vaø d2 cheùo nhau .
b) Chöùng minh raèng d1 song song vôùi maët phaúng (P) : 6x – 14y – z – 40 = 0
.Tính khoaûng caùch giöõa d1 vaø (P).
c) Tìm ñieåm N ñoái xöùng vôùi ñieåm M( 1 ; -1 ;0) qua ñöôøng thaúng d1.
Baøi 30 : Trong khoâng gian vôùi heä truïc toïa ñoâ Oxyz cho hình laäp phöông
ABCD.A’B’C’D’ . Bieát toïa ñoä caùc ñieåm A(0 ;0 ; 0) ,B(1 ; 0 ; 0 ) , D( 0 ; 1 ; 0)
vaø A’( 0 ; 0 ; 1) .
a) Haõy xaùc ñònh caùc ñieåm coøn laïi cuûa hình laäp phöông .
b) Goïi M,N laàn löôït laø trung ñieåm cuûa AB vaø B’C’ . Tính khoaûng caùch giöõa
MN vaø AD.
Baøi 31 : Trong khoâng gian vôùi heä toïa ñoä Oxyz , cho 4 ñieåm A( 2 ; 3 ;1), B( 1 ; 1 ; -1),
C(2 ; 1 ; 0) , D(0 ; 1 2) .
a. Chöùng minh A,B,C,D laø 4 ñænh cuûa töù dieän .
b. Tính khoaûng caùch giöõa hai ñöôøng thaúng AB vaø CD .
c. Vieát phöông trình ñöôøng thaúng AB .
d. Vieát phöông trình maët caàu coù taâm treân ñöôøng thaúng AB vaø qua hai ñieåm C vaø D.
Baøi32: Tính goùc giöõa :
 x = − 2 + 3t
x − 3 y + z = 0
a) d1 :  y = − 1 , d2 :  .
z = 4 − t x − y + z − 4 = 0

x −1 y + 2 z + 2
b) d: = = vaø (P): 3x + y – z +13 = 0
3 1 4
Baøi 33: Trong khoâng gian vôùi heä toïa ñoä Oxyz cho ñieåm M(-1 ;2 ;-3) vaø maët phaúng
(P):4x–y + 4z -15 = 0.
a) Tìm toïa ñoä ñieåm H laø hình chieáu vuoâng goùc cuûa M treân (P).
b) Tìm toïa ñoä ñieåm M’ ñoái xöùng vôùi M qua (P).
Baøi 34 :Tìm taâm vaø baùn kính cuûa caùc maët caàu sau :
a) x2 + y2 + z2 – 6x +2y – 4z – 2 = 0.
b) x2 + y2 + z2 – 4x +8y +2z – 4 = 0
67
Baøi 35 : Trong khoâng gian vôùi heä toïa ñoä Oxyz cho ñöôøng troøn (C) coù phöông trình :
 x 2 + y 2 + z 2 − 4 x − 6 y − 4 z − 32 = 0
 . Tìm taâm vaø baùn kính cuûa (C) .
 x + 4 y + 5 z + 18 = 0
Baøi 36 : Trong khoâng gian vôùi heä toïa ñoä Oxyz , vieát phöông trình maët caàu (S) ñi qua
3 ñieåm
A(1 ; 1 ; 0), B(-1 ; 1 ; 2) , C( 1 ; -1 ; 2) vaø coù taâm naèm treân maët phaúng (P): x + y
+z–4=0.
Baøi 37:Trong khoâng gian vôùi heä toïa ñoä Oxyz cho ñieåm I(1;-1;2) vaø maët phaúng
(P):3x+4y–z–23 = 0.
Vieát phöông trình maët caàu taâm I vaø tieáp xuùc vôùi (P) . Tìm toïa ñoä tieáp ñieåm .
Baøi 38 : Trong khoâng gian vôùi heä toïa ñoä Oxyz cho ba ñieåm A( 0 ; 0 ; 1) ,B(2 ; 1 ; 1) ,
C(1 ; 0 ; 0).
Vieát phöông trình ñöôøng troøn ngoaïi tieáp tam giaùc ABC .
Baøi 39 : Vieát phöông trình maët caàu (S) ngoaïi tieáp töù dieän ABCD vôùi A( 3 ; 2; 6 ), B(
3 ; -1 ; 0 ),
C( 0; -7 ; 3 ),D(-2 ;1 ; -1).
Baøi 40 : Trong khoâng gian vôùi heä toïa ñoä Oxyz cho döôøng thaúng d:
x −1 y + 2 z − 2
= = vaø hai ñieåm A( 0 ;1;-1) , B(2; -1 3).Vieát phöông trình maët caàu
2 1 1
(S) coù taâm treân d vaø (S) ñi qua hai ñieåm A,B
x y −1 z +1
Baøi 41: Cho ñöôøng thaúng (d): = = vaø hai maët phaúng(α): x+ y -2z +5 = 0
2 1 2
,
(β) :2x – y + z + 2 = 0 .Vieát phöông trình maët caàu coù taâm treân (d) vaø tieáp xuùc vôùi
hai maët phaúng (α),(β).

Chñ ®Ò Sè phøc
C¸c d¹ng to¸n th−êng gÆp
1, D¹ng 1 : Thùc hiÖn phÐp tÝnh (céng, trõ, nh©n, chia) c¸c sè phøc

68
1.1 C¸ch gi¶i tæng qu¸t :

Cho hai sè phøc z = a + bi , z ' = a '+b' i . Khi ®ã


1, Céng hai sè phøc : z + z ' = (a + a ') + (b + b')i
2, Trõ hai sè phøc : z − z ' = (a − a ') + (b − b')i
3, Nh©n hai sè phøc : z.z ' = (aa '−bb' ') + (ab'+ba ')i

4, Sè phøc liªn hîp, m« ®un :


-Sè phøc liªn hîp cña sè phøc z = a + bi lµ z = a − bi
- M« ®un cña sè phøc z = a + bi lµ z = a 2 + b 2
z'
5, Chia hai sè phøc : Chia hai sè phøc z’ cho z kh¸c 0 ta lµm nh− sau : = z '.z −1 víi
z
1 z' a − bi 1
z −1 = 2
z , tøc lµ : = (a '+b' i ) 2 2
= 2 [(a.a'+bb') + (ab'−a' b )i ]
z z a +b a + b2

1.2. VÝ dô. TÝnh:

( 2 − 3i )  12 +  2 − 15i 1 + i tan α


2


(
3i  ; c / 1 + 2i ; d / )
3 + 2i
; e/
1 − i tan α
.
a/ 5 + 2i – 3(-7+ 6i) ; b/
1.3 H−íng dÉn

a/ 5 + 2i – 3(-7+ 6i)=26-16i
 1 
( ) 1  1
( )
b/ 2 − 3i . + 3i  = 2. − 3. − 3 +  2. 3 +  − 3.  i = 4 + i
 3 3
2  2   2  2
c/ (1 + 2i ) . = 1 + 2.1.
2
2i + 2i 2 = −1 + 2 2i
2 − 15i 3 − 2i 1 1
d/ = (2 − 15i ) 2 2
= [(2.3 − 15.2 ) + (− 2.2 − 15.3)i ] = (− 24 − 49i )
3 + 2i 3 +2 13 13

L−u ý : Khi nh©n hai sè phøc víi nhau ta nh©n nh− nh©n ®a thøc víi ®a thøc víi chó ý
i2 =-1, i4=1
1.4 Bµi tËp tù luyÖn
TÝnh :
( ) ( )2
1/ a, 1 + 2i + i + (3 − 2i ) , b/ 3 + 2i , c/ 1 + 2i (3 − 2i ) , ( ) d/ (1 + i )i (3 − 2i ) ,
2 − 5i i 2 2 + 5i 2−i
e/ , f/ , g/ , h/ , k/
1 + 2i 1+ i 1 − 3i i 1− i
1 3 1
2/ Cho z = +
2 2
3
()
i . TÝnh z, , z 2 , z ,1 + z + z 2 , z 2 + z
z
()
2

2. D¹ng 2.TÝnh m«®un

69
2.1 C¸ch gi¶i tæng qu¸t
M« ®un cña sè phøc z = a + bi lµ z = a 2 + b 2

2.2 VÝ dô :
2
 1   3 
2
1 3
M«®un cña z = + i lµ z =   +  = 1.
2 2  2   2 

2.3 Bµi tËp tù luyÖn


T×m M«®un cña c¸c sè phøc sau
2 − 5i
( ) ( ) (
2
)
a, 1 + 2i , b/ 3 + 2i , c/ 1 + 2i (3 − 2i ) , d/ (3 − 2i ) , e/
1 + 2i
, f/
i
1+ i
3. D¹ng 3: C¨n bËc hai cña sè phøc`
3.1 C¸ch gi¶i tæng qu¸t

x 2 − y 2 = a
Sè phøc w = a + bi cã c¨n bËc hai lµ sè phøc z = x + yi nÕu w = z 2 ⇔ 
2 xy = b

3.2 VÝ dô : T×m c¨n bËc hai cña sè phøc : 3+4i

Gäi c¨n bËc hai cña sè phøc 3+4i lµ x+yi. khi ®ã ta cã


 2  2 2
x −   = 3  x 4 − 3x 2 − 4 = 0 x 2 = 4
x 2 − y 2 = 3   
x  
3 + 4i = ( x + yi )
2
⇔ ⇔ ⇔ 2 ⇔ 2
2 xy = 4  2 y = y =
 y = x  x  x

VËy sè phøc 3+4i cã hai c¨n bËc hai lµ : 2+i vµ -2-i

x 2 − y 2 = a
Chó ý : Khi t×m c¨n bËc hai cña sè phøc ta ph¶i gi¶i hÖ ph−¬ng tr×nh  , x, y lµ c¸c sè
2 xy = b
thùc
3.3 Bµi tËp tù luyÖn
T×m c¨n bËc hai cña mçi sè phøc sau :
a/ − 1 + 4 3i , b/ 4 + 6 5i , c/ − 1 + −2 6i , d/ 3 − 4i , e/ i, f/ -4
4. D¹ng 4: Ph−¬ng tr×nh bËc hai Az2 +Bz + C =0, A, B, C lµ c¸c sè phøc
4.1 C¸ch gi¶i tæng qu¸t

Ta cã ∆ = B 2 − 4 AC
* NÕu ∆ ≠ 0 ph−¬ng tr×nh cã hai nghiÖm ph©n biÖt
− B +δ − B −δ
z1 = , z2 = víi δ lµ mét c¨n
70 bËc hai cña ∆
2A 2A
−B
* NÕu ∆ =0 th× z = z =
4.2 VÝ dô: Gi¶i c¸c ph−¬ng tr×nh
a/ z 2 − z + 1 = 0 b/ z 2 + (− 2 + i )z − 2i = 0
a/ z 2 − z + 1 = 0 b/ z 2 + (− 2 + i )z − 2i = 0
4.3 H−íng dÉn
1 ± 3i
a/ ∆ = (− 1)2 − 4.1.1 = −3 ≠ 0 , nªn PT cã hai nghiÖm ph©n biÖt lµ :
2
b/ ∆ = (− 2 + i )2 − 4.1.(− 2i ) = −3 + 4i = (2 + i )2 ≠ 0 , nªn PT ®· cho cã hai nghÖm ph©n biÖt
2−i+2+i 2−i −2−i
z1 = = 2, z 2 = = −i
2 2
Chó ý : NÕu PT bËc hai cã hÖ sè lµ c¸c sè thùc th× nghiÖm lµ hai sè liªn hîp nhau
4.4 Bµi tËp tù luyÖn
Gi¶i c¸c ph−¬ng tr×nh sau
1. a/ z 2 − z − 1 = 0 , b/ z 2 + 2 z + 5 = 0 , c/ z 2 + (1 − 3i )z − 2(1 + i ) = 0 , (z 2 + i )(z 2 − 2iz − 1) = 0
1 1 1
2. a/ z + = 1 , b/ z + = 2 , c/ z + = 2i
z z z
3. T×m b, c sao cho ph−¬ng tr×nh : z + bz + c = 0 cã mét nghiÖm lµ : 1+i
2

5. D¹ng 5 : D¹ng l−îng gi¸c cña sè phøc

D¹ng l−îng gi¸c cña sè phøc z = a + bi lµ z = r (cos ϕ + i sin ϕ )


a b
víi r = a 2 + b 2 . cos ϕ = , sin ϕ =
r r
a b
TÝnh r = a 2 + b 2 , x¸c ®Þnh acgumen ϕ tho¶ m·n : cos ϕ = , sin ϕ =
r r

5.1 VÝ dô
a/ sè 2 cã d¹ng l−îng gi¸c lµ 2(cos0+isin0)
π
b/ Sè 1+i cã m«®un b»ng 2 vµ cã mét acgumen b»ng nªn d¹ng l−îng gi¸c lµ:
4
 π π
z = 2  cos + i sin 
 4 4
1 − 3
c/ Sè 1 − 3i cã m«®un b»ng 2 vµ cã mét acgumen tho¶ m·n cos ϕ = , sin ϕ = nªn chän
2 2
−π  −π −π 
ϕ= vµ d¹ng l−îng gi¸c cña 1 − 3i lµ z = 2 cos + i sin 
3  3 3 
d/ Sè : − (cos ϕ + i sin ϕ ) cã d¹ng l−îng gi¸c lµ (cos(ϕ + π ) + i sin (ϕ + π ))
e/ Sè : cos ϕ − i sin ϕ cã d¹ng l−îng gi¸c lµ cos(− ϕ ) + i sin (− ϕ )

71
Chó ý : ë d¹ng l−îng gi¸c cña c¸c sè phøc yªu cÇu r>0
5.2 Bµi tËp tù luyÖn :
ViÕt c¸c sè phøc sau d−íi d¹ng l−îng gi¸c
2 − 5i 1
(
a/ 1 − 3i , b/ 1 − i , c/ (1 − i ) 1 + 3i , c/) 1 + 2i
, e/ (1 − i )2i , f/
1 + 2i
, g/ i cos ϕ + sin ϕ

6. D¹ng 6 : øng dông cña c«ng thøc Moa-vr¬


C«ng thøc Moa-vr¬
[r (cos ϕ + i sin ϕ )]n = r n (cos n ϕ + i sin n ϕ )

6.1.
6.1. øng
ø dông t×m c¨n bËc hai cña sè phøc d−íi d¹ng l−îng gi¸c z = r (cos ϕ + i sin ϕ )

6.2. C¸ch gi¶i tæng qu¸t

Sè phøc ®· cho cã hai c¨n bËc hai d¹ng l−îng gi¸c lµ :


 ϕ ϕ  ϕ ϕ  ϕ  ϕ 
r  cos + i sin  vµ − r  cos + i sin  = r  cos + π  + i sin  + π  
 2 2  2 2  2  2 

6.3. VÝ dô
T×m c¨n bËc hai cña sè phøc 1-i
6.4 H−íng dÉn
 −π −π 
Ta cã 1-i = 2  cos + i sin  nªn c¨n bËc hai cña sè phøc 1-i lµ :
 4 4 
4  −π −π  4   π  π 
2  cos + i sin  vµ 2  cos π −  + i sin  π −  
 8 8    8  8 

6.5 Bµi tËp tù luyÖn


6
 1 
1/ TÝnh a/ (3 − i ) , b/ 
6

 1 + 3i 
1
2/ T×m c¨n bËc hai cña c¸c sè phøc sau : 3-4i, 4+3i, 1+i, 3, 4i,
2i + 1

Chñ ®Ò øng dông cña tÝch ph©n

I. DiÖn tÝch h×nh ph¼ng


1 D¹ng 1: DiÖn tÝch h×nh ph¼ng giíi h¹n bëi ®å thÞ hµm sè y= f(x) vµ c¸c ®−êng th¼ng x =a, x =
b, trôc hoµnh
72
1.1 C¸ch gi¶I tæng qu¸t:

b
DiÖn tÝch h×nh ph¼ng ®−îc x¸c ®Þnh bëi c«ng thøc: S = ∫ f ( x )dx (1)
a

§Ó tÝnh ®−îc tÝch ph©n (1) ta ph¶I thùc hiÖn c¸c b−íc sau:
B−íc1: Gi¶i PT: f(x) = 0 t×m thªm nghiÖm thuéc ®o¹n [a;b]( nÕu cã)
B−íc 2: XÐt dÊu f(x) trªn ®o¹n[a;b] ®Ó khö dÊu gi¸ trÞ tuyÖt ®èi
b
B−íc 3: TÝnh S = ∫ f ( x )dx
a

Sau khi khö dÊu gi¸ trÞ tuyÖt ®èi ta tÝnh tÝch ph©n b×nh th−êng, kÕt qu¶ thu ®−îc lµ diÖn tÝch cÇn
t×m.

1.2 VÝ dô:
a/ TÝnh diÖn tÝch h×nh ph¼ng giíi h¹n bëi ®å thÞ hµm sè y= x3 - 1 vµ c¸c ®−êng th¼ng x = 2, trôc
hoµnh
b/ TÝnh diÖn tÝch h×nh ph¼ng giíi h¹n bëi ®å thÞ hµm sè y= x3 - 1 vµ c¸c ®−êng th¼ng
x = 2 , trôc tung vµ trôc hoµnh
1.3 H−íng dÉn

b
DiÖn tÝch h×nh ph¼ng ®−îc x¸c ®Þnh bëi c«ng thøc: S = ∫ f ( x )dx (1)
a

2
2 2
 x4  11
a/ XÐt pt: x -1=0 ⇔ x = 1 . VËy S = ∫ x − 1dx = ∫ (
3 3
x − 1 dx = 
3
) − x  =
1 1  4 1 4
b/ DÔ thÊy x 3 − 1 ≤ 0 trªn [0;1], x 3 − 1 ≥ 0 trªn [1;2]. Do ®ã diÖn tÝch cña h×nh ph¼ng cÇn t×m lµ
1 2
2 1 2
 x4   x4  3 11 7
3
(
S = ∫ x − 1dx = ∫ 1 − x dx + ∫
3
) ( 3
)
x − 1 dx =  x −  + 
4 0  4
− x  = + =
0 0 1  1 4 4 2

Chó ý:
* NÕu bµi to¸n cho míi cã mét cËn th× ph¶i gi¶i: PT f(x) = 0 ®Ó t×m cËn thø hai vµ xÐt dÊu tÝch
b
ph©n ∫ f (x )dx
a

73
* NÕu bµi to¸n ch−a cho cËn th× ph¶i gi¶i: PT f(x) = 0 ®Ó t×m c¸c cËn vµ xÐt dÊu tÝch ph©n
b

∫ f (x )dx
a

1.4 Bµi tËp tù gi¶i:


1/ TÝnh diÖn tÝch h×nh ph¼ng giíi h¹n bëi ®å thÞ hµm sè y= sinx+1 , trôc hoµnh vµ c¸c ®−êng

th¼ng x= 0, x =
6
2/ TÝnh diÖn tÝch h×nh ph¼ng giíi h¹n bëi ®å thÞ hµm sè y= 4-x2 , trôc hoµnh vµ c¸c ®−êng th¼ng
x= 0, x = 3
3/ TÝnh diÖn tÝch h×nh ph¼ng giíi h¹n bëi ®å thÞ hµm sè y= cos2x , trôc hoµnh vµ c¸c ®−êng th¼ng
x= 0, x = π
4/ TÝnh diÖn tÝch h×nh ph¼ng giíi h¹n bëi ®å thÞ hµm sè y= 1-x2 , trôc hoµnh vµ c¸c ®−êng th¼ng
x= 0, x = 3
5/ TÝnh diÖn tÝch h×nh ph¼ng giíi h¹n bëi ®å thÞ hµm sè y= 4+x2 , trôc hoµnh vµ c¸c ®−êng th¼ng
x= 0, x = 3
2/ D¹ng 2: DiÖn tÝch h×nh ph¼ng giíi h¹n bëi ®å thÞ hµm sè y= f(x), y= g(x) vµ hai ®−êng th¼ng x
=a, x = b
2.1 C¸ch gi¶I tæng qu¸t

b
DiÖn tÝch h×nh ph¼ng ®−îc x¸c ®Þnh bëi c«ng thøc: S = ∫ f ( x ) − g (x )dx (1)
a

§Ó tÝnh ®−îc tÝch ph©n (1) ta ph¶I thùc hiÖn c¸c b−íc sau:
B−íc1: T×m nghiÖm cña ph−¬ng tr×nh f(x)= g(x) trªn ®o¹n [a;b] (nÕu cã)
B−íc 2: XÐt dÊu f(x) – g(x) trªn ®o¹n[a;b] ®Ó khö dÊu gi¸ trÞ tuyÖt ®èi
b
B−íc 3: TÝnh ∫ f (x ) − g (x )dx
a

Sau khi khö dÊu gi¸ trÞ tuyÖt ®èi ta tÝnh tÝch ph©n b×nh th−êng, kÕt qu¶ thu ®−îc lµ diÖn tÝch cÇn
t×m.

2.2. VÝ dô

74
a/ TÝnh diÖn tÝch h×nh ph¼ng giíi h¹n bëi ®å thÞ hµm sè y= x3 - 4x, trôc hoµnh, ®−êng th¼ng x = -2
vµ ®−êng th¼ng x = 4.
b/ TÝnh diÖn tÝch h×nh ph¼ng giíi h¹n bëi ®å thÞ hµm sè y= 2-x2 vµ ®−êng th¼ng y = -x

b
DiÖn tÝch h×nh ph¼ng ®−îc x¸c ®Þnh bëi c«ng thøc: S = ∫ f ( x ) − g (x )dx
a

x = 0
XÐt PT: x − 4 x = 0 ⇔  x = −2
3

 x = 2

DiÖn tÝch h×nh ph¼ng giíi h¹n bëi ®å thÞ hµm sè y= x3 - 4x, trôc hoµnh, ®−êng th¼ng
x = -2 vµ ®−êng th¼ng x = 4 lµ
4 0 2 4

∫ x 3 − 4 x dx = ∫ (x ) ( )
− 4 x dx + ∫ 4 x − x 3 dx + ∫ x 3 − 4 x dx ( )
3
S=
−2 −2 0 2

0 2 4
 x4   x4   x4 
=  − 2 x 2  +  2 x 2 −  +  − 2 x 2  = 4 + 4 + 32 = 40 (§VDT)
 4  −2  4 0  4 2
b
b/ DiÖn tÝch h×nh ph¼ng ®−îc x¸c ®Þnh bëi c«ng thøc: S = ∫ f ( x ) − g (x )dx
a

 x = −1
XÐt ph−¬ng tr×nh 2 − x 2 = − x ⇔ 
x = 2

DiÖn tÝch h×nh ph¼ng giíi h¹n bëi ®å thÞ hµm sè y= 2-x2 vµ ®−êng th¼ng y = -x lµ
2
2 2
 x2 x3  9
∫ (2 − x ) − (− x )dx = ∫ ( )
2 + x − x dx =  2 x + −  = (§VDT)
2 2
S=
−1 −1  2 3  −1 2

2.3 H−íng dÉn


2.4 Bµi tËp tù luyÖn:
1/ TÝnh diÖn tÝch h×nh ph¼ng giíi h¹n bëi
a/ §å thÞ hai hµm sè y = x , y = 3 x
b/ §å thÞ hai hµm sè y = 2 x 2 , y = x 4 − 2 x 2 trong miÒn x ≥ 0
c/ §å thÞ hai hµm sè y = x 2 − 4, y = − x 2 − 2 x vµ c¸c ®−êng th¼ng th¼ng x = -3 vµ x = -2
d/ §å thÞ hai hµm sè y = x 2 − 4, y = − x 2 − 2 x

Chó ý: *Khi tÝnh diÖn tÝch h×nh ph¼ng giíi h¹n bëi ®å thÞ hµm sè y= f(x), y= g(x) vµ ®−êng th¼ng x
=a, x = b ta cÇn gi¶i PT f = g ®Ó t×m c¸c nghiÖm thuéc ®o¹n [a;b]
*Khi tÝnh diÖn tÝch h×nh ph¼ng giíi h¹n bëi ®å thÞ hµm sè y= f(x), y=g(x) ta cÇn gi¶i PT: f=g ®Ó lÊy
cËn cho tÝch ph©n.

75
II. ThÓ tÝch cña vËt thÓ
1. ThÓ tÝch vËt thÓ sinh bëi h×nh ph¼ng giíi h¹n bëi ®å thÞ hµm sè y= f(x), trôc hoµnh vµ hai
®−êng th¼ng x =a, x = b khi quay quanh trôc hoµnh
1.1 C¸ch gi¶I tæng qu¸t

b
ThÓ tÝch cña vËt thÓ ®−îc tÝnh bëi c«ng thøc: V = π ∫ f 2
(x )dx
a

Nh− vËy ®Ó t×m thÓ tÝch ta chØ viÖc tÝnh tÝch ph©n trªn.

1.2 VÝ dô:
Cho h×nh ph¼ng A giíi h¹n bëi c¸c ®−êng y= x2, y=0, x= 0, x= 2. TÝnh thÓ tÝch cña khèi
trßn xoay t¹o thµnh khi quay h×nh A quanh Ox.

1.3 H−íng dÉn


2
2
 x5  32
V = π ∫ x dx = π 
4
 = π
0  5 0 5

1.4 Bµi tËp tù luyÖn:


1/ Cho h×nh ph¼ng A giíi h¹n bëi c¸c ®−êng y= xex/2, y=0, x= 0, x= 1. TÝnh thÓ tÝch cña khèi trßn
xoay t¹o thµnh khi quay h×nh A quanh Ox
2/ Cho h×nh ph¼ng A giíi h¹n bëi c¸c ®−êng y= cosx, y=0, x= 0, x = π / 4 . TÝnh thÓ tÝch cña khèi
trßn xoay t¹o thµnh khi quay h×nh A quanh Ox
3/ Cho h×nh ph¼ng A giíi h¹n bëi c¸c ®−êng y= x3, y=0, x= 0, x= 2. TÝnh thÓ tÝch cña khèi trßn
xoay t¹o thµnh khi quay h×nh A quanh Ox
4/ Cho h×nh ph¼ng A giíi h¹n bëi c¸c ®−êng y=x1/2ex/2, y=0, x= 0, x= 1. TÝnh thÓ tÝch cña khèi trßn
xoay t¹o thµnh khi quay h×nh A quanh Ox

Chñ ®Ò Hµm sè ph©n thøc bËc hai trªn

76
bËc nhÊt
C¸c d¹ng to¸n th−êng gÆp
ax 2 + bx + c
1. D¹ng 1: Kh¶o s¸t sù biÕn thiªn vµ vÏ ®å thÞ hµm sè y = , (a ≠ 0 )
a ' x + b'
1.2 Quy tr×nh bµi to¸n KSHS
Tu©n theo c¸c b−íc kh¶o s¸t hµm sè bËc nhÊt trªn bËc nhÊt
x 2 + 2x + 6
1.3. VÝ dô Kh¶o s¸t sù biÕn thiªn vµ vÏ ®å thÞ hµm sè y =
x −1
1.4 H−íng dÉn
1.TËp x¸c ®Þnh: R\{1}
2.Kh¶o s¸t sù biÕn thiªn
a, Giíi h¹n v« cùc, giíi h¹n t¹i v« cùc, tiÖm cËn
x 2 + 2x + 6 x 2 + 2x + 6
lim y = lim+ = +∞ , lim y = lim− = −∞
x →1+ x →1 x −1 x →1− x →1 x −1

VËy ®−êng th¼ng x = 1 lµ tiÖm cËn ®øng cña ®ß thÞ hµm sè khi x → 1+ vµ khi x → 1−
x 2 + 2x + 6 x 2 + 2x + 6
lim y = lim = +∞ , lim y = lim = −∞
x → +∞ x → +∞ x −1 x → −∞ x → −∞ x −1
x 2 + 2x + 6 9
TiÖm cËn xiªn: Ta cã = x +3+
x −1 x −1
9 9
lim ( y − x − 3) = lim = 0 , lim ( y − x − 3) = lim =0
x → +∞ x → +∞ x − 1 x → −∞ x → −∞ x − 1

VËy d−êng th¼ng y = x +3 lµ tiÖm cËn xiªn cña ®å thÞ hµm sè khi x → +∞ vµ khi
x → −∞

b, ChiÒu biÕn thiªn


9
* Ta cã y ′ = 1 −
(x − 1)2
9
= 0 ⇔ ( x − 1) = 9 ⇔ x = −2, x = 4
2
y′ = 0 ⇔ 1 −
(x − 1) 2

* B¶ng biÕn thiªn


x −∞ -2 1 4 +∞
y’ + 0 - - 0 +
y -2 +∞

−∞ −∞ 10
Hµm sè ®ång biÕn trªn mçi kho¶ng ( − ∞ ; -2) vµ (4; + ∞ )
77
Hµm sè nghÞch biÕn trªn mçi kho¶ng (-2;1) vµ (1;4)
Hµm sè ®¹t cùc ®¹i t¹i x = -2, GT cùc ®¹i y = -2
Hµm sè ®¹t cùc tiÓu t¹i x = 4, GT cùc tiÓu y = 10
3.§å thÞ
§å thÞ hµm sè c¾t trôc tung t¹i ®iÓm (0;-6)
§å thÞ hµm sè kh«ng c¾t trôc hoµnh
§å thÞ hµm sè nhËn ®iÓm I(1;4) lµm t©m ®èi xøng

15

x2+2⋅x+6
f( x) =
x-1
g ( x) = x+3
10

20 10 10

10

1.5 Bµi tËp tù gi¶i:


Kh¶o s¸t sù biÕn thiªn vµ vÏ ®å thÞ cña hµm sè
x 2 + 2x + 1 x 2 − 2x + 2 x 2 − 2x + 4 − 2x 2 − 2x
a, y = ; b, y = ; c, y = ; d, y = ;
x −1 x −1 x−2 x+2
2 2 1 4
e, y = 2 x + 1 + ; f, y = 2 x − 1 + ; g, y = x + 1 + ; h, y = x +
x−2 x+2 x−2 x
C¸c d¹ng to¸n liªn quan t−¬ng tù nh− hµm sè bËc nhÊt trªn bËc nhÊt
Chó ý: hµm sè bËc hai trªn bËc nhÊt lu«n cã tiÖn cËn ®øng vµ tiÖm cËn xiªn

2. D¹ng 2: Bµi to¸n t×m gi¸ trÞ lín nhÊt, Gi¸ trÞ nhá nhÊt.
78
2.1. Ph−¬ng ph¸p chung

Ph−¬ng ph¸p 1: lËp b¶ng biÕn thiªn nÕu bµi to¸n t×m GTLN,GTNN trªn kho¶ng(a;b)
Ph−¬ng ph¸p 2: NÕu bµi to¸n t×m GTLN, GTNN trªn ®o¹n [a;b] th× lµm theo c¸c b−íc
B−íc 1: TÝnh f’(x)
B−íc 2: T×m nghiÖm x1, x 2 cña PT f’(x) = 0 trªn ®o¹n [a;b]
B−íc3: So s¸nh f(a), f(x1), f(x2), f(b)
B−íc 4: Sè lín nhÊt trong c¸c sè trªn lµ GTLN, sè bÐ nhÊt trong c¸c sè trªn lµ GTNN

2.2 VÝ dô:
x 2 − 2x + 4
a/ T×m GTLN, GTNN cña hµm sè f ( x ) = trªn ®o¹n [3;7]
x−2
2
b/ T×m GTNN cña hµm sè y = 2 x − 1 + trªn ko¶ng (-3/2;7)
x+2
2.3 H−íng dÉn
4 x 2 − 4x x 2 − 4x  x = 0 ∉ [3;7]
a/ Ta cã: f ' ( x) = 1 − = f ' ( x) = 0 ⇔ =0⇔  x = 4 ∈ [3;7]
( x − 2) 2 ( x − 2) 2 ( x − 2) 2 

Ta cã f(3) = 7, f(7) = 39/5, f(4) = 6.


VËy GTLN cña hµm sè trªn lµ: 39/5, GTNN c¶u hµm sè trªn lµ 6
2 x 2 + 4x + 3
b/ Ta cã: f ' ( x) = 2 − = 2 .
( x + 2) 2 ( x + 2) 2

 −3 
 x = −1 ∈  2 ;7 
 
2
x + 4x + 3
f ' ( x) = 0 ⇔ =0⇔
( x + 2) 2
  3 
 x = −3 ∉  − ;7 
  2 

B¶ng biÕn thiªn


x -3/2 -1 7

f’(x) - 0 +

f(x)
-1

Dùa vµo b¶ng biÕn thiªn ta suy ra GTNN cña hµm sè b»ng -1 khi x= -1

2.4 Bµi tËp tù gi¶i:


79
1.T×m GTLN, GTNN cña c¸c hµm sè.
x 2 + 2x + 1 x 2 − 2x + 2
a, y = trªn ®o¹n [0;7] b, y = trªn ®o¹n [3/2;6]
x −1 x −1
x 2 − 2x + 4 − x 2 − 2x + 1
c, y = trªn ®o¹n [-3;1] d, y = trªn ®o¹n [-5;-5/2]
x−2 x+2
2.T×m GTNN cña c¸c hµm sè .
2 1
a, y = 2 x + 1 + trªn kho¶ng (2;6) b, y = x + 1 + trªn kho¶ng (2;6)
x−2 x−2
4
c, y = x + trªn kho¶ng (0;6)
x

Chñ ®Ò HÖ ph−¬ng tr×nh mò

80
vµ hÖ ph−¬ng tr×nh logarit

1. Ph−¬ng ph¸p chung

B−íc 1: t×m §KX§ cho hÖ ph−¬ng tr×nh


B−íc 2: §−a hÖ ph−¬ng tr×nh vÒ d¹ng kh«ng cßn mò hoÆc l«garit
B−íc 3: TiÕn hµnh gi¶i

 y 5 x 2 −51x +10 = 1
1.1 VÝ dô: Gi¶i hÖ ph−¬ng tr×nh 
 xy = 5

1.2 H−íng dÉn

(1) t−¬ng ®−¬ng víi TH1Vµ TH2

y =1
TH1: 
 xy = 5
5 x 2 − 51x + 10 = 0
TH2: 
 xy = 5
Gi¶I c¸c hÖ ë TH1 vµ TH2 lµ c¸c hÖ ®¹i sè th«ng th−êng.

1.3 Bµi tËp tù gi¶i


Gi¶i c¸c hÖ ph−¬ng tr×nh sau
 x y 2 −7 y +10 = 1  x x 2 − y 2 −16 = 1  x + y = 2
1.  2.  3.  2
 x + y = 8 (x > 0 )  x − y = 2 (x > 0 ) ( y + 1) x + x + 2 = 1
3 − x .2 y = 1152 log x y = 2  y − log 3 x = 1
4.  5.  6. 
log x +1 ( y + 23) = 3
12
log 5 ( x + y ) = 2 y
x = 3
3 − x .2 y = 1152 l o g ( x 2 + y 2 ) = 1 + l o g8 3 .2 = 972
x y
4)  7.  8. 
log 5 ( x + y ) = 2 l o g ( x + y ) − l o g ( x − y ) = l o g 3 log 3 ( x − y ) = 2

log 2 (u + v ) − log 2 (u − v ) = 1 0,5 log 2 x − log 2 y = 0


9.  10. 
2 2 2 2
u − v = 2 x − 5 y + 4 = 0
x + y = 1 5 3 x .2 y = 200 2 x + 2 y = 5

9)  x y
11.  3 12. 
2 − 2 = 2 5 2 x + 2 2 y = 689 2 x + y = 4

81

You might also like